Está en la página 1de 70

Matemáticas Discretas I

por Marı́a Luisa Pérez Seguı́


Introducción
Se presenta aquı́ el material correspondiente a un primer curso en Matemáticas Discretas,
el cual se imparte en la Facultad de Ciencias Fı́sico-Matemáticas de la Universidad Michoa-
cana. El material del libro constituyó el 100 % del curso impartido el semestre de agosto de
2015 a enero de 2016 en la Facultad. Parte fundamental del aprendizaje de las Matemáticas
Discretas está basado en la resolución de problemas. Por esta razón, se intercalan numerosos
ejemplos inmediatamente después de que se ha introducido un concepto nuevo, de manera
que puedan practicarse las técnicas de resolución. Se proponen también diversos ejercicios;
algunos de ellos son rutinarios, mientras que la solución de otros requiere de un mayor es-
fuerzo, imaginación y dedicación. Buena parte del material fue una adecuación de los dos
libros de Combinatoria de la autora, los cuales se utilizan intensamente en etrenamientos
para la Olimpiada de Matemáticas en todo el paı́s.
La sección 1 proporciona el material básico del conteo. Se tratan aquı́ las fórmulas de
permutaciones y combinaciones. Se explican también las técnicas de multiplicación y las de
separación en casos.
La sección 2 estudia el Teorema del Binomio.
En la sección 3 se prueban varias fórmulas importantes en la Combinatoria. El método
usado es el de comparaciones, que consiste en contar de dos (o más) formas distintas lo
mismo.
En la sección 4 se explica el método de demostración por inducción. Éste es un método
básico en un gran número de demostraciones de matemáticas discretas y tiene aplicaciones
y métodos de aplicación muy diversos. Se dan ejemplos de todos estos métodos y se explican
cuidadosamente los errores que comete en general un principiante.
En la sección 5 se tratan algunos ejemplos en los que la idea es contar caminos con ciertas
condiciones. El tema se presta para practicar técnicas de conteo.
La sección 6 provee otro método de conteo importante, consistente en sumar y después
compensar restando cuando hubieran aparecido repeticiones.
La sección 7 es una muy breve introducción a la probabilidad matemática. Simplemente
se establecen las bases y se estudian algunos ejemplos.
En la sección 8 se estudia un principio muy básico y, a primera vista, elemental, que es el
Principio de las Casillas, y que simplemente establece que si se quieren colocar en cajas un
número de objetos mayor que el de cajas, entonces al menos una caja deberá tener más de un
objeto. Las aplicaciones de este principio son muy diversas y algunas de ellas son bastante
complicadas. La resolución de problemas de esta ı́ndole abre un panorama interesante en el
razonamiento matemático.
La sección 9 estudia el tema de coloración, el cual simplemente consiste en distinguir
unos objetos de otros; dicha clasificación ofrece nuevas técnicas de conteo.

i
Finalmente, en la sección 10 se da una introducción al importante tema de las gráficas
(o grafos). Este tema es esencial en el estudio de la computación. Por esta razón, se trata
de abarcar los principales conceptos y resultados más básicos, incluyendo paseos eulerianos
y ciclos hamiltonianos, planaridad y la fórmula de Euler, y coloración de mapas.

Marı́a Luisa Pérez Seguı́


Fac. Cs. Fı́sico-Matemáticas
Universidad Michoacana de San Nicolás de Hidalgo
diciembre, 2015.

ii
Índice
Introducción I

1. Conteo 1

2. Teorema del binomio 12

3. Comparaciones 14

4. Inducción Matemática 16

5. Caminos 28

6. Inclusión y Exclusión 31

7. Probabilidad Combinatoria 34

8. Principio de las Casillas 41

9. Coloración 46

10.Gráficas 50
10.1. Paseos eulerianos y ciclos hamiltonianos . . . . . . . . . . . . . . . . . . . . 55
10.2. Gráficas planas . . . . . . . . . . . . . . . . . . . . . . . . . . . . . . . . . . 59

iii
1. Conteo

Uno de los conceptos matemáticos abstractos más primitivos que conocemos es el de


número y, dentro de los números, el de los números naturales enteros positivos: 1, 2,
3, 4, etc. Con ellos representamos las cantidades de objetos que se nos presentan en la vida
cotidiana. En esta sección desarrollaremos algunas técnicas que permiten determinar con
facilidad cantidades. Comencemos por ilustrar la necesidad de aprender estas técnicas de
conteo con unos ejemplos: Si se nos enseña un puñado de canicas y se nos pregunta cuántas
son, un vistazo nos bastará para contarlas y dar la respuesta; sin embargo si se nos pregunta
cuántas patas tienen 100 perros, en lugar de buscar los 100 animales y contarles las patas,
haremos una abstracción, y la operación: 4 × 100 = 400 nos dirá la respuesta; utilizamos aquı́
una técnica muy simple de multiplicación. Desde luego hay preguntas que necesitan técnicas
más elaboradas. Estudiaremos estas técnicas mediante ejemplos que iremos complicando
gradualmente.

Analicemos primero con cuidado un ejemplo que a primera vista es trivial pero que nos
enseña la clave básica del conteo.

1.1 Ejemplo. ¿Cuántos números enteros de tres o menos cifras hay?


Solución. La respuesta a esta pregunta es fácil: Hay 1000 pues son todos los números
enteros del 0 al 999. Esta solución no nos enseña gran cosa. Retomemos ahora el problema
buscando una solución constructiva; esto es, para cualquier n = 1, 2, 3, . . ., la cantidad de
números de hasta n + 1 cifras se puede obtener de la cantidad de números de hasta n cifras:
simplemente se multiplica por 10. Vamos a describir con detalle este procedimiento:
Números de a lo más una cifra hay 10, a saber, 0, 1, 2, 3, 4, 5, 6, 7, 8 y 9. Para contar los
de hasta dos cifras (del 0 al 99) no necesitamos escribirlos todos; basta con observar que la
primera cifra puede ser cualquiera de los 10 dı́gitos 0, 1, 2, 3, 4, 5, 6, 7, 8, 9, y por cada uno de
éstos hay 10 terminaciones distintas; por ejemplo, los números de dos cifras que empiezan con
4 son: 40, 41, 42, 43, 44, 45, 46, 47, 48 y 49, diez en total; lo mismo para cada una de las otras
decenas. Ası́ la cantidad de enteros entre 0 y 99 es 10×10 = 100. El siguiente paso es análogo:
Para contar los números de hasta tres cifras hay que agregar un dı́gito (posiblemente 0) a
cada uno de los 100 números de 2 o menos cifras; como hay diez posibilidades la respuesta
será 10 × 100 = 1000. ♦

Este procedimiento de “construir sobre lo ya construido” que hemos utilizado se llama


procedimiento inductivo . Muchas demostraciones de propiedades y fórmulas de números
naturales se basan en él. Más adelante se estudiará esto con detalle. El principio combinatorio
que manejamos en el ejemplo anterior (y que manejaremos en los siguientes) es:

1.2. Principio Fundamental de Conteo. Si una cierta tarea puede realizarse de m


maneras diferentes y, para cada una de esas formas, una segunda tarea puede realizarse de
n maneras distintas, entonces las dos tareas juntas pueden realizarse (en ese orden) de mn

1
formas diferentes.

1.3 Ejemplo. ¿Cuántas palabras de tres letras se pueden formar si se dispone de un


alfabeto con dos letras: a y b. (Nota: Son permisibles palabras como bba.)
Solución. Procederemos como en el ejemplo anterior. En este caso conviene ilustrarlo
haciendo un “diagrama árbol”:

Resolvamos ahora el ejemplo utilizando nuestro Principio Fundamental de Conteo. Con-


sideremos tres casillas: , la primera para la letra inicial, la segunda para la letra central
y la tercera para la letra final. En cada casilla hay dos elecciones posibles: la letra a o la
letra b. La respuesta es entonces 2 × 2 × 2 = 8. El procedimiento inductivo es como sigue:
En la primera casilla hay 2 posibilidades para elegir la letra. Una vez formada una palabra
de una letra: a o b, para agrandarla a una palabra de dos letras hay dos posibilidades, ası́
que palabras de dos letras hay 2 × 2 = 4. Para completar cada una de éstas a una palabra
de tres letras hay dos posibilidades; entonces hay 4 × 2 = 8 palabras de tres letras. ♦

1.4 Ejemplo. ¿Cuántas placas distintas hay con dos letras a la izquierda y tres números
a la derecha? (Nota: Consideraremos el alfabeto de 27 letras castellanas.
Solución. Seguimos el procedimiento de las casillas del ejemplo anterior:

27 × 27 × 10 × 10 × 10 = 729 000. ♦
| {z } | {z }
lugares lugares
para letras para números

1.5 Ejemplo. ¿Cuántas banderas bicolores se pueden formar si se dispone de 4 lienzos


de tela de colores distintos y un asta? (Nota: Banderas como rojo-rojo no son permisibles;
por otro lado, es importante el color que queda junto al asta, de esta manera banderas como
rojo-azul y azul-rojo se consideran distintas.)

2
Solución. En este caso consideramos dos casillas. La de la izquierda, digamos, representa
el lienzo junto al asta, el cual tiene 4 elecciones posibles. Una vez elegido éste, el color para
la derecha se puede escoger de 3 formas (pues no se permite la repetición de colores). Ası́
hay 4 × 3 = 12 formas distintas de formar las banderas. ♦

1.6 Ejercicio. Escribir todas las banderas que pueden formarse según el ejemplo an-
terior si los colores son rojo (R), azul (A), verde (V ) y blanco (B).

1.7 Ejemplo. Misma pregunta que en el ejemplo anterior pero ahora suponiendo que
no hay asta. (En este caso no habrá distinción entre las banderas rojo-azul y azul-rojo.)
Solución. Para resolver este ejemplo analicemos la respuesta del ejemplo anterior. En
aquél, en la colección total de las 12 banderas posibles podemos aparear cada bandera con
su opuesta; por ejemplo la bandera azul-verde la apareamos con la bandera verde-azul. Cada
una de las del ejemplo anterior se esta contando dos veces y, por tanto, la respuesta es
12
2
= 6. ♦

1.8 Ejercicio. En el resultado del ejercicio 1.6 aparear cada una de las banderas con
su opuesta. Dar una lista de 6 banderas que ilustre la respuesta del ejemplo 1.7.

1.9 Ejemplo. ¿De cuántas formas se pueden sentar 5 personas en 5 sillas numeradas
del 1 al 5?
Solución. En el asiento #1 se puede sentar cualquiera de las 5 personas; para cada elección
de la primera persona, la segunda puede ser cualquiera de las 4 restantes; ası́ en las dos
primeras sillas el número de elecciones posibles es 5 × 4 = 20. Continuamos de manera
análoga. Para simplificar dibujemos 5 casillas simbolizando los 5 asientos. Sobre cada casilla
escribamos el número respectivo de posibilidades y multipliquemos:
5 × 4 × 3 × 2 × 1 = 120. ♦

Si n es un número natural, el producto de todos los números naturales del 1 al n aparece


muy frecuentemente en problemas de combinatoria; se llama n factorial o factorial de n
y se denota por n!. (Ası́ la respuesta del ejemplo 1.9 es 5! = 120.)

Alejándose de la interpretación de n! como el producto de los naturales de 1 a n, se define


0! = 1;
esto permite incluir el caso n = 0 en algunas fórmulas en las que interviene n!. Entonces
0! = 1
1! = 1
2! = 1×2=2
3! = 1×2×3=6
4! = 1 × 2 × 3 × 4 = 24.

3
Es fácil darse cuenta que el número 5 del ejemplo 1.9 y el que sean personas y asientos en
lugar de cualquier otra cosa no es relevante; podemos generalizarlo como sigue:

El número Pn de distintas formas en que se pueden ordenar n objetos es n!. Cada una de
las listas ordenadas que se forman con los n objetos se llama permutación (de los objetos).
Tenemos entonces que el número de permutaciones de n objetos es Pn = n!.

1.10 Ejemplo. De un grupo de 5 estudiantes quiere elegirse una comisión de 3 para


que cada uno visite un museo de una lista de 3 museos. ¿Cuántas comisiones distintas se
pueden formar?
Solución. Utilizando el esquema de casillas (cada una representando un museo) como
arriba, tenemos que el resultado es

5 × 4 × 3 = 60. ♦

1.11 Ejemplo. De un grupo de 5 estudiantes quiere elegirse una comisión de 3 para


que juntos visiten un museo (el mismo todos). ¿Cuántas comisiones diferentes se pueden
formar?
Solución. Hay que observar que la diferencia entre este ejemplo y el anterior es que no
importa el orden en la elección. En el ejemplo anterior habı́a distición entre las casillas pues
cada una representaba un museo en particular distinto a los otros; en éste no hay distinción
entre las casillas pues, por ejemplo, una comisión en que se haya elegido la sucesión de
alumnos Ana-Beto-Carlos se considerará igual a la sucesión Beto-Carlos-Ana y también
igual a la sucesión Ana-Carlos-Beto. Nuestro interés es entonces determinar en la cantidad
5 × 4 × 3, en cuántas sucesiones aparece el mismo conjunto de alumnos. Para responder
esto conviene plantear esta parte del ejemplo al revés: Consideremos un conjunto fijo de 3
personas, por ejemplo el formado por Ana (A), Beto (B) y Carlos (C) y contemos de cuántas
formas se pueden ordenar estos 3. Observemos que el número de formas es precisamente el
número de permutaciones de las 3 personas, o sea, P3 = 3! = 6. Entonces cada grupo de 3
personas se está contando 6 veces en el producto 5 × 4 × 3, ası́ que la respuesta al ejemplo
será
5×4×3
= 10. ♦
3!

1.12 Ejercicio. En los ejemplos 1.10 y 1.11 supongamos que el grupo de los 5 alumnos
está formado por Ana (A), Beto (B), Carlos (C), Daniel (D) y Elena (E). Hacer la lista de
los 60 arreglos de estos alumnos en los que se elige 3 para visitar museos distintos, agrupando
en esa lista las colecciones que resultan iguales si todos van a un mismo museo.

En el ejemplo anterior aprendimos el siguiente principio:

1.13. El número de colecciones (en las que el orden no importa) con r elementos que

4
se pueden seleccionar dentro de un conjunto de n elementos (n ≥ r ≥ 1) es
n × (n − 1) × · · · × (n − (r − 1))
.
r!

Este número recibe el nombre de combinaciones de n en r y se denota por nr . Dicho




de otra manera, el número de subconjuntos de r elementos que tiene un conjunto con n
elementos es nr . (En el ejemplo 1.11, n = 5 y r = 3 y la respuesta es 53 .) Nótese que la
fórmula 1.13 no tiene sentido para n = 0; sin embargo sı́ tiene sentido hablar del número
de subconjuntos con 0 elementos dentro de un conjunto con n elementos; sabemos que este
número es 1 pues sólo hay un conjunto sin elementos que es el llamado conjunto vacı́o.
Definimos entonces  
n
= 1.
0

1.14 Ejercicio. Sea X = {a, b, c, d, e}. Escribir todos los subconjuntos de X con
(a) 0 elementos,
(b) 1 elemento,
(c) 2 elementos,
(d) 3 elementos,
(e) 4 elementos y
(f) 5 elementos.
Verificar que en cada caso el número de subconjuntos obtenido sea 5r y que el número


total de subconjuntos sea 25 = 32.

1.15 Ejercicio. Basándose en la interpretación de nr como el número de subconjuntos




de r elementos dentro de un conjunto con n elementos, explicar por qué


   
n n
= .
r n−r
1.16 Ejercicio. Calcular 72 , 75 , 55 y 94 .
   

Con la intención de simplificar la fórmula 1.13 sobre las combinaciones de n en r, ob-


servemos que, para 1 ≤ r ≤ n − 1, el numerador se puede “completar” a n! multiplicando
por (n − r)!; si lo “completamos” deberemos compensar dividiendo también por (n − r)!.
Tendremos entonces que para r = 1, 2, . . . , n − 1,
1.17.  
n n!
= .
r r!(n − r)!

Recordemos que se ha definido 0! = 1 y n0 = 1; notemos entonces que si sustituimos




r = 0 (y, posiblemente también n = 0) en el lado derecho de la fórmula 1.17 obtendremos

5
n! n!
0!n!
= 1. De la misma manera, al sustituir r = n obtendremos n!0!
= 1. Ası́, también en estos
casos extremos vale la fórmula 1.17.

1.18 Ejercicio. Volver a hacer los ejercicios 1.15 y 1.16 utilizando la fórmula 1.17.

1.19 Ejemplo. De un grupo de 10 niños y 15 niñas se quiere formar una colección de


5 jóvenes que tenga exactamente 2 niñas. ¿Cuántas colecciones  distintas se pueden formar?
15 15×14
Solución. La elección de las 2 niñas se puede hacer de 2 = 2! = 105 formas. Como
deben ser 5 en total y debe haber 2 niñas exactamente, entonces los niños serán 3; éstos se
pueden escoger de 10 3
= 10×9×8
3!
= 120 formas. Por tanto el resultado es 105×120 = 12 600. ♦

Como hemos visto, al determinar cantidades buscamos simplificar nuestras cuentas uti-
lizando “homogeneidades” en el problema. Con este propósito, en algunas ocasiones es con-
veniente dividir en casos de manera que en cada uno de ellos haya homogeneidad, y después
sumar las respuestas. Un ejemplo muy simple de esto serı́a el siguiente: Si tenemos 4 paquetes
de 100 hojas de papel y otros 3 paquetes de 200 hojas cada uno, entonces el número total
de hojas que tenemos es
4 × 100 + 3 × 200 = 1000.

Comparemos el siguiente ejemplo con el anterior, tomando en cuenta la búsqueda de


homogeneidades, como acabamos de decir.

1.20 Ejemplo. De un grupo de 10 niños y 15 niñas se quiere formar una colección de


5 jóvenes que tenga a lo más 2 niñas. ¿Cuántas colecciones distintas se pueden formar?
Solución. Vamos a resolver este ejemplo como el anterior pero separando por casos y
después sumando las respuestas de cada uno de los casos.
Caso 1: Que la colección tenga 2 niñas exactamente: 15
 10
2
= 12 600.
 3
15 10
Caso 2: Que la colección tenga exactamente 1 niña: 1 4 = 3 150.
Caso 3: Que la colección no tenga niñas: 15
 10
0 5
= 252.
La respuesta al ejemplo es 12 600 + 3 150 + 252 = 16 002. ♦

1.21 Ejemplo. Un grupo de 15 personas quiere dividirse en 3 equipos de 5 personas


cada uno. Cada uno tendrá una labor especı́fica distinta a las demás. ¿De cuántas formas
distintas es posible hacer la distribución?
Solución. Escojamos uno por uno los equipos. La elección del primer equipo puede hacerse
15

de 5 = 3 003 formas; para elegir el segundo equipo ya sólo habrá 10 personas de dónde
escoger, por tanto éste se podrá elegir de 10

5
= 252 formas. El tercer equipo quedará formado
automáticamente con la elección de los otros dos. Entonces el número de formas de hacer la
elección sucesiva es 3 003 × 252 × 1 = 756 756. ♦

1.22 Ejemplo. Un grupo de 15 personas quiere dividirse en 3 equipos de 5 personas

6
cada uno. Todos los equipos tendrán la misma labor. ¿De cuántas formas es posible hacer la
distribución?
Solución. En este caso no hay distinción entre los equipos ası́ que hay que dividir el
resultado del ejemplo anterior entre 3!, que es el número de permutaciones de los equipos.
La respuesta es entonces 126 126. ♦

1.23 Ejemplo. En una bolsa hay 3 pelotas rojas y 2 azules. Se quiere formar una fila
con todas ellas. ¿De cuántas maneras distintas puede quedar la fila?
Solución. Primera forma. Consideremos todas las permutaciones de las 5 pelotas y con-
temos cuántas de esas permutaciones son indistinguibles entre sı́. Las permutaciones de las
5 pelotas sabemos que son 5! = 120. En cualquiera de las permutaciones fijémonos en la
ubicación de las pelotas rojas; por ejemplo − roja − roja roja. éstas pueden revolverse
entre sı́ (3! veces) formando colecciones indistinguibles, y lo mismo ocurre con las del otro
color. Vamos a explicar lo anterior con más detalle: Denotemos las pelotas rojas por R1 , R2
y R3 , y las azules por A1 y A2 . Entonces las siguientes listas (en las que se han permutado
las rojas pero se han dejado fijas las azules) representan la misma colección:
 
A1 R1 A2 R2 R3
A1 R1 A2 R3 R2 
 
A1 R2 A2 R1 R3 
A1 R2 A2 R3 R1  .
 
 
A1 R3 A2 R1 R2 
A1 R3 A2 R2 R1

Estas 3! listas deben considerarse como una sola. Además, en cada una de ellas también se
pueden revolver las azules entre sı́ (2! permutaciones). Entonces al considerar las permuta-
ciones de las 5 pelotas, cada arreglo se está contando 3! × 2! = 12 veces en lugar de 1. La
5!
respuesta al ejemplo es pues 3!2! = 10.
Segunda forma. Primero podemos contar las posibilidades para colocar las pelotas rojas
en los 5 lugares disponibles; esto nos dará la elección de 3 lugares, que puede hacerse de
5
3
= 10 maneras. Para colocar las 2 azules ya sólo sobran 2 lugares ası́ que esto se puede
2

hacer de 2 = 1 forma. El resultado es 10 × 1 = 10. ♦

1.24 Ejercicio. Escrı́banse las 10 filas distintas que se pueden formar con las pelotas
en el ejemplo 1.23.

1.25 Ejemplo. En una bolsa hay 3 pelotas rojas y 2 azules. ¿Cuántas filas distintas
de 3 pelotas se pueden formar?
Solución. Como son 5 pelotas en total pero sólo se van a considerar filas de 3, hay que dejar
dos pelotas sin colocar. Consideraremos los distintos casos por separado y después sumaremos
3!
las respuestas parciales. Si las dos pelotas que quedan fuera son rojas, hay 1!2! = 3 arreglos
3! 3!
con las restantes. Análogamente hay 3! = 1 fila que deja las 2 pelotas azules fuera, y 2!1! =3

7
filas que dejan una azul y una roja fuera. La respuesta al ejemplo es 3 + 1 + 3 = 7. ♦

1.26 Ejercicio. Escribir los 7 arreglos de pelotas del ejemplo 1.25 .

En algunas ocasiones, para poder hacer bien las cuentas, nuestra búsqueda de homoge-
neidad nos lleva a que es más fácil contar lo opuesto de lo que queremos y después restar de
un total. Ilustramos esto con el siguiente ejemplo.

1.27 Ejemplo. ¿De cuántas maneras pueden ordenarse en un estante 3 cuadernos


rojos, 4 azules y 2 verdes, si los verdes no deben quedar juntos?
Solución. Conviene contar primero todas las ordenaciones posibles y después restar aquéllas
en las que los verdes quedan juntos. El número total de filas (incluyendo aquéllas en que los
9!
verdes quedan juntos es 3!4!2! = 1260. Para contar las que tienen juntos los cuadernos verdes
pensemos éstos como pegados formando un solo cuaderno; ahora determinemos el número de
8!
arreglos con 3 cuadernos rojos, 4 azules y 1 verde; éste es 3!4! = 280. La respuesta al ejemplo
es 1260 − 280 = 980. ♦

1.28. Los ejemplos siguientes se refieren a la baraja usual de pókar: Cada carta tiene
un sı́mbolo llamado número que puede ser cualquiera de los 13 sı́mbolos siguientes: A, 2,
3, 4, 5, 6, 7, 8, 9, 10, J, Q o K, y otro sı́mbolo llamado palo que puede ser cualquiera de
los 4 siguientes: ♠ (espada), ♥ (corazón), ♦ (diamante) o ♣ (trébol). Todos los palos
se combinan con todos los números para formar la baraja completa con 13 × 4 = 52 cartas
como se ilustra a continuación:

A♥ 2♥ 3♥ 4♥ 5♥ 6♥ 7♥ 8♥ 9♥ 10♥ J♥ Q♥ K♥
A♦ 2♦ 3♦ 4♦ 5♦ 6♦ 7♦ 8♦ 9♦ 10♦ J♦ Q♦ K♦
A♠ 2♠ 3♠ 4♠ 5♠ 6♠ 7♠ 8♠ 9♠ 10♠ J♠ Q♠ K♠
A♣ 2♣ 3♣ 4♣ 5♣ 6♣ 7♣ 8♣ 9♣ 10♣ J♣ Q♣ K♣
Se llama mano de pókar cualquier colección de 5 cartas de la baraja. La siguiente nomen-
clatura es usual:
par: dos cartas del mismo número.
tercia: tres cartas del mismo número.
pókar: cuatro cartas del mismo número.
full: una tercia y un par.
flor: cinco cartas del mismo palo.
corrida: cinco cartas con numeración consecutiva (según el orden en que se escribieron
arriba, pero permitiendo A también como número final, en seguida de K).
Observemos que el número total de manos de pókar es 52

5
= 2 598 960.

1.29 Ejemplo. ¿Cuántas manos de pókar tienen tercia exactamente (es decir, que no
sea full ni pókar).

8
Solución. Primera forma. Ponemos 5 casillas: las tres primeras para la tercia y las otras
dos para las otras cartas. La primera carta se puede escoger arbitrariamente; la segunda sólo
tiene 3 posibilidades pues debe tener el mismo número que la primera; la tercera ya sólo
puede ser elegida de 2 maneras distintas; como no importa el orden de estas 3 cartas, este
número deberá dividirse entre 3!. La cuarta carta se debe escoger dentro de las 48 que son de
número distinto al de la tercia. Para la quinta carta ya sólo sobran 44 cartas pues el número
debe ser también distinto. La cuarta y quinta pueden haberse escogido en cualquier orden
por lo que se deberá dividir entre 2!.
52 × 3 × 2 48 × 44
× = 54 912.
| {z3! } 2! }
| {z
tercia cartas distintas

Segunda forma. También formamos primero la tercia pero eligiendo antes el número que le
corresponderá: Tenemos 13 números para escoger y, una vez escogido el número, las 3 cartas
que forman la tercia deben escogerse dentro de 4 posibles; entonces el número de tercias
es 13 43 . Para escoger las otras dos cartas utilizando este mismo método razonamos como


sigue: Hay que escoger 2 números (pues queremos que las otras 2 cartas sean de números
distintos) dentro de los 12 que sobran; esta elección se puede hacer entonces de 12
2
formas.
En cada uno de estos números que se hayan elegido hay que escoger 1 carta, cosa que puede
hacerse de 41 formas. El resultado escrito en esta forma es
    2
4 12 4
13 × ,
3 2 1

que, desde luego, también es igual a 54 912. ♦

1.30 Ejemplo. ¿Cuántas manos de pókar tienen dos pares (distintos) exactamente?
Solución. Procedemos como en el ejemplo 1.29.

Primera forma.
1er par 2o par
z }| { z }| {
52 × 3 48 × 3
2! 2! × 44 = 123 552.
2!
(Nota: Hay que dividir entre 2! porque no importa el orden entre los dos pares.)

Segunda forma.   2
13 4
× 44 = 123 552. ♦
2 2

1.31 Ejemplo. ¿Cuántas manos de pókar tienen corrida?

9
Solución. El número más bajo de la corrida puede ser cualquiera de los siguientes: A,
2, 3, 4, 5, 6, 7, 8, 9 o 10, que son 10 posibilidades. Pongamos 5 casillas; la primera casilla
será para la carta de número menor, la siguiente casilla será para el siguiente número, y ası́
sucesivamente hasta la quinta casilla que será para la carta con el número mayor. Una vez
escogido el número menor para la corrida, todos los demás números quedan determinados y
lo único que falta escoger es el palo. Entonces la cantidad de corridas es 10×4×4×4×4×4 =
10 240. ♦

1.32 Ejercicio. (∗) ¿De cuántas maneras diferentes se pueden ordenar 8 personas
alrededor de una mesa redonda? (Nota: Dos distribuciones se considerarán iguales si una se
puede obtener de la otra mediante un giro.)

1.33 Ejercicio. (∗) ¿De cuántas maneras distintas se pueden sentar 5 personas en una
fila de 8 asientos numerados del 1 al 8?

1.34 Ejercicio. (∗) ¿Cuántas diagonales tiene un polı́gono regular de n lados?

1.35 Ejercicio. (∗) Probar la Fórmula de Pascal:


     
n+1 n n
= + ,
r+1 r r+1
para r y n números enteros con 0 ≤ r < n.

1.36 Ejercicio. El Triángulo de Pascal está definido como el triángulo de números


en el que el renglón número n aparecen los n + 1 números
         
n n n n n
, , ,··· , , .
0 1 2 n−1 n
Se muestran a continuación los primeros 4 renglones del Triángulo de Pascal. Utilizar la
fórmula del ejercicio anterior para construir los 10 primeros renglones.
1 1
1 2 1
1 3 3 1
1 4 6 4 1
1.37 Ejercicio. De un grupo de 8 jóvenes se quiere escoger un equipo de volibol con 6
integrantes y otro de basquetbol con 5 integrantes (deberán tener jugadores en común) ¿De
cuántas maneras es posible escoger los equipos?

1.38 Ejercicio. (∗) De un grupo de 24 personas se quiere elegir 5 representantes de la


siguiente forma: Pedro y Luis deben estar en el grupo elegido. Hay 8 mujeres en total pero a
lo más deben figurar 2 en el grupo. ¿De cuántas maneras distintas puede hacerse la elección?

10
1.39 Ejercicio. (∗) De un grupo de 30 socios de un club se quiere elegir una mesa
directiva con un presidente, un secretario y 3 equipos de 2 personas cada uno. ¿Cuántas
mesas directivas distintas se pueden formar?

1.40 Ejercicio. (∗) ¿Cuántas palabras distintas se pueden escribir revolviendo las le-
tras de la palabra COMBINATORIA?

1.41 Ejercicio. (∗) De un conjunto de 10 botes de distintos colores se quiere escoger 5


de tal manera que 3 sean para dulces y 2 sean para chocolates. ¿De cuántas formas distintas
es posible hacer la elección?

1.42 Ejercicio. (∗) Se dispone de una colección de 30 pelotas divididas en 5 tamaños


distintos y 6 colores diferentes de tal manera que en cada tamaño hay los 6 colores. ¿Cuántas
colecciones de 4 pelotas tienen exactamente 2 pares de pelotas del mismo tamaño (que no
sean las 4 del mismo tamaño)?

1.43. El siguiente problema se refiere al conjunto usual de 28 fichas de dominó en que


cada ficha muestra dos números de la colección 0, 1, 2, 3, 4, 5 y 6 (posiblemente repetidos),
como esquematizamos a continuación:

6 6 6 5 6 4 6 3 6 2 6 1 6

5 5 5 4 5 3 5 2 5 1 5

4 4 4 3 4 2 4 1 4

3 3 3 2 3 1 3

2 2 2 1 2

1 1 1

Se llaman fichas dobles aquéllas en que los dos números mostrados son iguales. Se llama
mano de dominó cualquier colección de 7 de las 28 fichas. Nótese que el número total de
manos de dominó es 28

7
= 1 184 040.

1.44 Ejercicio. (∗) ¿Cuántas manos de dominó tienen por lo menos 2 fichas dobles?

11
2. Teorema del binomio

El siguiente es un resultado muy importante en aritmética. Lo probaremos aquı́ utilizando


algunas de las técnicas de combinatoria que hemos aprendido. Más adelante volveremos a
probarlo usando el método de inducción.

2.1 Teorema. Teorema del Binomio de Newton. Sean a y b números arbitrarios


y sea n un número natural. Entonces
       
n n n n n−1 n n−r r n n
(a + b) = a + a b + ··· + a b + ··· + b .
0 1 r n
Demostración. La expresión (a + b)n significa que tenemos que multiplicar a + b consigo
mismo n veces. Entonces, al desarrollar todo el producto, los términos que obtenemos están
dados por todas las posibles elecciones de los números a o b en cada uno de los n factores
(por ejemplo, (a + b)3 = (a + b)(a + b)(a + b) = aaa + aab + aba + abb + baa + bab + bba + bbb =
a3 + 3a2 b + 3ab2 + b3 ). Observemos entonces que los términos obtenidos son de la forma as br ,
con 0 ≤ s, r ≤ n y s + r = n, es decir, s = n − r. Ahora notemos que an−r br aparece cada
vez que se eligió b en r de los factores y a en el resto, ası́ que el número de veces que aparece
este término es nr . Al agrupar términos semejantes tenemos la fórmula deseada. ♦

Como hemos visto, los números nr (para 0 ≤ r ≤ n) aparecen como coeficientes en




la expansión de un binomio elevado a la potencia n; por esta razón reciben el nombre de


coeficientes binomiales. En los ejercicios 1.35 y 1.36 vimos que para una n elegida no muy
grande podemos obtener fácilmente los coeficientes binomiales sin recurrir en cada caso a la
n n!

fórmula r = (n−r)!r! .

2.2 Ejercicio. Utilizar el Teorema del Binomio para desarrollar la expresión (a − 2b)5 .

2.3 Ejercicio. (∗) Utilizar el Teorema del Binomio para desarrollar la expresión (a +
2b − 2c )4 .

2.4 Ejercicio. (∗) Encontrar el coeficiente del término a7 b4 ce2 en el desarrollo de (a +


b + c + d + e)14 . (Sugerencia: Proceder como en la prueba del Teorema del Binomio.)

2.5 Ejercicio. (∗) Utilizar el Teorema del Binomio para probar la fórmula
       
n n n n
+ + + ··· + = 2n .
0 1 2 n

12
2.6 Ejercicio. (∗) Utilizar el Teorema del Binomio para probar la fórmula
           
n n n n n n
+ + + ··· = + + ··· .
0 2 4 1 3 5

¿Qué interpretación se puede dar a esta fórmula en términos de subconjuntos de un conjunto?

2.7 Ejercicio. (∗) Probar que para cualquier número natural se tiene la fórmula
 2  2  2  2  
n n n n 2n
+ + + ··· + = .
0 1 2 n n

(Sugerencia: Examinar el coeficiente de xn al desarrollar ambos miembros de la igualdad


(1 + x)2n = (1 + x)n (1 + x)n .)

2.8 Ejercicio. (∗) Encontrar el término que no contiene a x en el desarrollo de


9


1
x +√
4x
.

13
3. Comparaciones

En esta sección veremos algunos ejemplos en los que el contar de dos maneras diferentes
lo mismo nos conduce a obtener ciertas fórmulas en combinatoria (que a su vez pueden usarse
para resolver otros problemas).

3.1 Ejemplo. Probar la fórmula de Gauss


n(n + 1)
1 + 2 + 3 + ··· + n = .
2
Solución. Contaremos las colecciones de 2 elementos que pueden escogerse dentro de
un conjunto de n + 1 elementos de dos maneras diferentes. La comparación de los dos re-
sultados nos demostrará la veracidad de la fórmula. Consideremos ası́ el conjunto X =
{x1 , x2 , . . . , xn+1 }. Pongamos los subconjuntos de X que tienen dos elementos en una lista,
como sigue:
{x1 , x2 }, {x1 , x3 }, {x1 , x4 }, · · · {x1 , xn+1 },
{x2 , x3 }, {x2 , x4 }, · · · {x2 , xn+1 },
{x3 , x4 }, · · · {x3 , xn+1 },
..
.
{xn−1 , xn+1 }.
De esta lista es fácil observar que el número de subconjuntos de X con 2 elementos es
precisamente lo que aparece del lado izquierdo en la igualdad que queremos probar. Por otro
lado, sabemos que el número de subconjuntos de 2 elementos que tiene un conjunto con n + 1
elementos es n+1
2
, que es precisamente lo que aparece en el miembro derecho de la igualdad,
y ası́ queda completa la demostración. ♦

3.2 Ejemplo. Probar la fórmula


       
n n n n
+ + + ··· + = 2n .
0 1 2 n
Solución. Probaremos que cada uno de los miembros de la igualdad en la fórmula cuenta el
número de subconjuntos que tiene un conjunto de n elementos. Esto es claro para el miembro
izquierdo. Para probar que también el miembro derecho representa lo mismo, una vez más,
contemos otra cosa que es equivalente: el número de sucesiones de longitud n que se pueden
formar con dos sı́mbolos S y N , es decir el número de “palabras” de n letras que pueden
formarse con los sı́mbolos S y N . Obviamente, este número es 2n , que es el miembro derecho
de la igualdad buscada, ası́ que, para completar nuestra prueba, bastará que probemos que
el número de sucesiones mencionado es también el número de subconjuntos. Para lograrlo,
establezcamos una correspondencia entre las sucesiones y los subconjuntos de manera que
cada sucesión represente un subconjunto y viceversa. Esto lo podemos hacer observando que
para determinar un subconjunto hay que ir tomando uno a uno los elementos del conjunto
total, e ir diciendo si el elemento pertenece (S) o no (N ) al subconjunto. Para que esto quede

14
más claro, ilustremos el “apareo”de sucesiones con subconjuntos en el caso del conjunto
X = {a, b, c} (n = 3):
{a, b, c} ↔ SSS
{a, b} ↔ SSN
{a, c} ↔ SN S
{b, c} ↔ N SS
{a} ↔ SN N
{b} ↔ N SN
{c} ↔ N N S
{ } ↔ N N N. ♦

Es interesante también notar que en el ejemplo anterior, al probar que el número de


subconjuntos de un conjunto con n elementos es 2n , establecimos una comparación (en este
caso, correspondencia uno a uno) entre los subconjuntos y las sucesiones de longitud n que
constan de los sı́mbolos S y N .

3.3 Ejercicio. Probar la fórmula de 3.2 usando la forma de contar palabras de cierta
longitud con cierto número de letras.

3.4 Ejercicio. Probar la fórmula de Pascal 1.35 utilizando comparaciones.

3.5 Ejercicio. Probar que si n ∈ N entonces


       
n−1 n n n n
n2 =1· +2· +3· + ··· + n · .
1 2 3 n

3.6 Ejercicio. (∗) Probar que si m, n y r son naturales con 0 ≤ r ≤ m, n, entonces


             
m+n m n m n m n m n
= + + + ··· + .
r 0 r 1 r−1 2 r−2 r 0

3.7 Ejercicio. (∗) Contar el número de ternas de números del 1 al n (n natural) que
están en orden creciente de dos maneras distintas para probar la fórmula
       
n 2 3 n−1
= + + ··· + .
3 2 2 2

15
4. Inducción Matemática

En esta sección recordaremos qué significa hacer una demostración o construcción por
inducción.

4.1 Ejemplo. Analicemos la sucesión (lista) de números n2 + n para n natural. El


primer término de nuestra lista es 2, pues cuando n = 1, n2 + n = 12 + 1 = 2; el segundo
término es 6 ya que 22 + 2 = 6. Ası́ obtenemos la sucesión:

2, 6, 12, 20, 30, 42, 56, 72, 90, . . .

Podemos notar que todos los términos que escribimos son pares. ¿Es cierto que todos los
términos de la sucesión son pares?
Solución. La respuesta es sı́. Podemos probar esto directamente (sin usar inducción ma-
temática), observando que para cualquier natural n, el número n2 + n se puede escribir
como n(n + 1), o sea que todos los términos de la sucesión son el producto de dos enteros
consecutivos y, como uno de los dos enteros debe ser par, el producto también lo será.
Más abajo haremos otra demostración utilizando el método de inducción, pero primero
hablemos un poco sobre el procedimiento que seguiremos:
Notemos que con la sola proposición: “Para cualquier natural n, el número n2 +n es par”,
estamos abarcando una infinidad de proposiciones (una para cada n): 12 + 1 es par, 22 + 2
es par, 32 + 3 es par, etc. Si tratamos de probar cada una individualmente no llegaremos
muy lejos; en cambio, si probamos (I1 ) que la primera proposición es cierta y (I2 ) que,
cada vez que todas las proposiciones anteriores a una fija P sean verdaderas también lo es
la misma P, entonces podemos concluir que todas las proposiciones son ciertas. En efecto,
comprobemos por ejemplo que de nuestro método (probar (I1 ) e (I2 )) se deduce que la 4a
proposición es cierta: La 1a proposición es cierta por (I1 ); utilizando esto tenemos que, por
(I2 ), la 2a proposición también es cierta; pero entonces, al tener que la primera y la segunda
afirmaciones son ciertas, por (I2 ) deducimos que la 3a proposición es verdadera; ahora ya
tendemos que la primera, la segunda y la tercera son ciertas ası́ que, otra vez usando (I2 )
concluimos que la 4a proposición también es válida. Ası́ como llegamos a la 4a proposición,
a cualquiera podemos llegar en un número finito de pasos, ası́ que, con sólo demostrar (I1 )
e (I2 ), podemos afirmar que todas las proposiciones son ciertas.
Probemos entonces (I1 ) e (I2 ) en nuestro ejemplo.
Demostración de (I1 ). Tenemos que 12 + 1 = 2, que es par.
Demostración de (I2 ). Supongamos que k ≥ 2 y que todas las afirmaciones desde la pri-
mera hasta la k-ésima (es decir, la que se encuentra en el lugar k) son verdaderas. Queremos
utilizar esta suposición para probar que, en este caso, también será verdadera la (k + 1)-ési-
ma. De hecho en nuestra demostración utilizaremos sólo la validez de la k-ésima (es decir, no
requeriremos utilizar toda la fuerza de nuestra suposición). El que la k-ésima afirmación sea
cierta nos dice que tomamos como verdadero el que k 2 + k sea par y queremos usar esto para
probar que (k + 1)2 + (k + 1) también es par. Desarrollemos la expresión (k + 1)2 + (k + 1)

16
para poder compararla con k 2 + k:

(k + 1)2 + (k + 1) = k 2 + 2k + 1 + k + 1 = (k 2 + k) + 2k + 2 = (k 2 + k) + 2(k + 1).

De esta manera hemos logrado expresar (k + 1)2 + (k + 1) como suma de dos números pares,
a saber k 2 + k (que estamos suponiendo es par) y 2(k + 1) (que es par por tener el número
2 como factor). Como la suma de números pares también es par, (k + 1)2 + (k + 1) es par,
como querı́amos probar. Esto termina la demostración de (I2 ).
Concluimos, entonces, que para todo número natural n, el número n2 + n es par. ♦

En el método de demostración por inducción se necesita un punto de partida al que se


le llama base de la inducción o, en forma abreviada, (BI). La suposición de que todas
las proposiciones anteriores a una dada son verdaderas se llama hipótesis de inducción,
abreviado (HI). La base de inducción puede constar de la demostración de una o de varias
afirmaciones. También la hipótesis de inducción puede constar de la suposición de que varias
afirmaciones anteriores a la que se va a demostrar sean verdaderas. La práctica nos dirá qué
tan fuerte necesitamos hacer nuestra hipótesis de inducción y cuántas afirmaciones deberán
tomarse como base de inducción.

Una forma de ilustrar por qué el método de inducción proporciona una demostración
correcta de algunas proposiciones es la siguiente: Supongamos que se tiene una hilera de
fichas de dominó colocadas de manera tal que cada vez que una caiga empujará a la siguiente
para que también caiga; si una persona empuja la primera ficha, podremos afirmar que cada
una de las fichas deberá caer en algún momento. sin embargo, notemos que es necesario que
las fichas estén lo suficientemente cerca (para que cada una empuje a la siguiente).

La forma en que uno hace ver cómo la validez de una proposición (o varias proposiciones)
“empuja(n)” la validez de la siguiente depende del problema particular de que se trate; a
veces la prueba puede ser muy sencilla y otras muy complicada.

Por otro lado, el que una persona no pueda demostrar satisfactoriamente un resultado
por inducción, no quiere decir nada sobre la validez del resultado; puede ser simplemente que
la sucesión de afirmaciones no tenga una liga tal que la validez de cada afirmación “empuje”
la validez de la siguiente. Siguiendo la analogı́a de las fichas de dominó supongamos que
las fichas de dominó estuvieran alejadas entre sı́ pero que de todas formas se cayeran por
otra razón (por ejemplo porque colocáramos un ventilador con suficiente fuerza frente a
ellas). También la práctica nos dirá en qué tipo de proposiciones podemos intentar hacer
una demostración por inducción y en cuáles no.

Es importante también aclarar que la hipótesis de inducción debe abarcar la base de


inducción; es decir, la primera afirmación que se suponga verdadera en la hipótesis de induc-
ción debe haber quedado demostrada independientemente en la base. También es importante
hacer notar que en cualquier demostración por inducción hay un paso comparativo en el
que se establece la relación o liga que existe entre una afirmación y la(s) precedente(s).

17
En resumen, para hacer una demostración por el método de inducción matemática se
deberán seguir los siguientes tres pasos:
Primer paso. Identificar la sucesión de proposiciones que abarca la proposición general
que se va a demostrar.
Segundo paso. Identificar y probar la base de inducción.
Tercer paso. Hacer una hipótesis de inducción (suposición de que todas las proposiciones
que preceden a una proposición fija son verdaderas) abarcando la base de inducción, y utilizar
esa suposición (o parte de ella), para probar que la proposición fija también es cierta. (Para
ello debe haberse hecho una comparación entre la afirmación fija que se va a demostrar y
la(s) anterior(es)).

4.2 Ejemplo. Probar que n = 4, 5, 6, . . . implica 2n < n!.


Solución. La sucesión de proposiciones es:
1a proposición: 24 < 4!.
2a proposición: 25 < 5!.
3a proposición: 26 < 6!.
4a proposición: 27 < 7!.
..
.

La base de inducción consiste en demostrar la 1a afirmación. Esto es sencillo ya que


24 = 16, 4! = 24 y 16 < 24, ası́ 24 < 4!.
La hipótesis de inducción puede ser, en este caso, la siguiente: Para cierta k ≥ 4 se tiene
k
2 < k!. (Notemos que la primera afirmación que se toma como cierta en esta hipótesis
es para k = 4 y que ésta quedó demostrada en la base de inducción.) Ahora usaremos la
hipótesis de inducción para hacer ver que 2k+1 < (k + 1)!. En efecto, esto se deduce de la
siguiente cadena de igualdades y desigualdades en la que en la primera desigualdad se usó
la hipótesis de inducción y en la segunda desigualdad se utilizó que k + 1 > 2 (esto último
es porque k ≥ 4):

2k+1 = 2 × 2k < 2 × k! < (k + 1) × k! = (k + 1)!.

(Notemos aquı́ que las dos igualdades en la cadena fueron de tipo comparativo: sirvieron para
establecer la liga entre la afirmación que estaba por probarse y la anterior, que se suponı́a
cierta según la hipótesis de inducción.) Hemos completado satisfactoriamente los tres pasos
en el método de inducción, ası́ que el resultado queda probado. ♦

4.3 Ejemplo. Probar por inducción la fórmula de Gauss

n(n + 1)
1 + 2 + 3 + ··· + n = ,
2
para n natural.

18
Solución. Nótese que el miembro izquierdo de la fórmula indica que dado el número
natural n hay que sumar todos los naturales más chicos que n, incluyendo el mismo n. Ası́,
la sucesión de proposiciones es:
1a proposición: 1 = 1×2
2
.
2 proposición: 1 + 2 = 2×3
a
2
.
3 proposición: 1 + 2 + 3 = 3×4
a
2
.
4a proposición: 1 + 2 + 3 + 4 = 4×5
2
.
..
.

En este caso la base de la inducción consiste en demostrar la 1a proposición, la cual es


obvia.
Tomaremos como hipótesis de inducción la siguiente: Para cierta k ≥ 1 (abarcando
la BI) se tiene que 1 + 2 + 3 + · · · + k = k(k+1)
2
. Queremos usar esto para probar que
(k+1) (k+1)+1
1 + 2 + 3 + · · · + (k + 1) = 2
. Para ello tomamos el lado izquierdo de la igualdad
que queremos probar y buscamos la forma de acomodar los términos para usar la hipótesis
de inducción y después obtener el lado derecho de la igualdad:

1 + 2 + 3 + · · · + (k + 1) = 1 + 2 + 3 + · · · + k + (k + 1)
= k(k+1)
2
+ (k + 1)
k(k+1)+2(k+1)
. = 2
= (k+2)(k+1)
2
.

Notamos que la primera igualdad es el paso comparativo y en la segunda igualdad se usó la


HI. Esto termina la demostración. ♦

Con inducción podemos también probar fórmulas en que hay más de una variable. El
siguiente es un ejemplo tı́pico. El contenido de la fórmula es muy útil en diversos problemas.

4.4 Ejemplo. Si r es cualquier número distinto de 1 (no necesariamente natural),


entonces
rn+1 − 1
1 + r + r2 + · · · + rn = .
r−1
para cualquier natural n.

19
Solución. La sucesión de proposiciones está indicada por n:
1+1
1a proposición: 1 + r = r r−1−1 .
r2+1 −1
2a proposición: 1 + r + r2 = r−1
.
3+1
3a proposición: 1 + r + r2 + r3 = r r−1−1 .
4+1
4a proposición: 1 + r + r2 + r3 + r4 = r r−1−1 .
..
.
Para probar la primera afirmación (base de la inducción) recordemos que (r + 1)(r − 1) =
r2 − 1 y dividamos esta ecuación por r − 1.
k+1
La HI en este caso es: “Para cierta k ≥ 1 se cumple 1 + r + r2 + · · · + rk = r r−1−1 .” A
partir de esta suposición probemos la fórmula correspondiente para n = k + 1:

1 + r + · · · + rk+1 = 1 + r + · · · + rk + rk+1

rk+1 −1
= r−1
+ rk+1 (por HI)

rk+1 −1+(r−1)rk+1
= r−1

rk+1 −1+rk+2 −rk+1


= rr−1

rk+2 −1
= r−1

r(k+1)+1 −1
= r−1
.

De esta serie de igualdades concluimos que, si la fórmula se supone válida para n = k,


entonces también lo será para n = k + 1, y con esto completamos satisfactoriamente todos
los pasos en el método de inducción. Concluimos entonces que la fórmula es cierta para todo
n. ♦

4.5 Ejercicio. Probar la afirmación del ejemplo anterior en forma no inductiva. (Su-
gerencia: Utilizar la misma idea con la que probamos la base de inducción.)

4.6 Ejercicio. Hacer una prueba inductiva y otra no inductiva de que el producto de
tres enteros consecutivos es múltiplo de 6.

4.7 Ejercicio. (∗) Hacer una prueba inductiva y otra no inductiva de la siguiente
fórmula para n natural:
1 1 1 n
+ + ··· + = .
1×2 2×3 n × (n + 1) n+1

20
1 1 1
(Sugerencia: Para la prueba no inductiva, observar que k(k+1)
= k
− k+1
.)

En los siguientes ejemplos compararemos algunas definiciones no recursivas con otras


recursivas.

4.8 Ejemplo. Definamos la sucesión a1 , a2 , a3 , . . . recursivamente por a1 = 1 y, para


n ≥ 2, an = an−1 + 2. Encontrar los primeros 6 términos de la sucesión y dar una definición
no recursiva de ella.
Solución. Para obtener los primeros 6 términos de la sucesión partimos de la base a1 = 1
y vamos construyendo los siguientes términos sumando 2 al término recién construido: 1, 3,
5, 7, 9, 11. Una definición no recursiva de la misma sucesión es an = 2n − 1. ♦

El ejemplo anterior es un caso particular de las llamadas sucesiones o progresiones


aritméticas; en general una sucesión aritmética es una sucesión de números a1 , a2 , a3 , . . .
en que la diferencia entre dos términos consecutivos cualesquiera es un número constante d,
es decir an+1 = an + d, para toda n.
Otros ejemplos de sucesiones aritméticas son:
1, 2, 3, 4, 5, . . . (aquı́ d = 1 y a1 = 1),
2, 4, 6, 8, 10, . . . (aquı́ d = 2 y a1 = 2),
10, 17, 24, 31, 38, . . . (aquı́ d = 7 y a1 = 10).
0, − 21 , −1, − 23 , −2, − 52 , . . . (aquı́ d = −1
2
y a1 = 0).

Hemos definido sucesión aritmética por recursión, es decir, en forma inductiva; si sólo
contamos con esto, es de esperar que cualquier afirmación sobre una sucesión aritmética
utilice inducción; el siguiente ejemplo (demostrado por inducción), permitirá trabajarlas en
forma no recursiva; en particular el resultado nos dice cómo conocer cualquier término de la
sucesión sin necesidad de conocer el anterior.

4.9 Ejemplo. Sea a1 , a2 , a3 , . . . una sucesión aritmética con diferencia d (es decir, para
toda n, an+1 = an + d. Probar que para n ≥ 2 se tiene an = a1 + (n − 1)d.
Solución. La sucesión de afirmaciones es:
1a afirmación: a2 = a1 + (2 − 1)d.
2a afirmación: a3 = a1 + (3 − 1)d.
3a afirmación: a4 = a1 + (4 − 1)d.
4a afirmación: a5 = a1 + (5 − 1)d.
..
.
La base de inducción es, en este caso, la primera afirmación (es decir, la afirmación
para n = 2). Es fácil darse cuenta de la validez de esta proposición pues, por definición,
a2 = a1 + d. Hagamos ahora la hipótesis de inducción: “Para cierta k ≥ 2 es verdad que
ak = a1 + (k − 1)d”. Utilizando esta HI probemos que también es cierto el resultado para

21
n = k + 1:
ak+1 = ak + d (por definición)
= a1 + (k − 1)d + d (por HI)
= a1 + kd.
Esto termina la demostración. ♦

4.10 Ejercicio. (∗) Una sucesión o progresión geométrica con razón r es una sucesión
de números a1 ,a2 ,. . . en que cada uno se obtiene del anterior multiplicando por el número r (es
decir, para n ≥ 2, an = an−1 r). Dar una definición no recursiva para la sucesión geométrica
con a1 = 5 y r = 21 . Calcular la suma de los primeros 100 términos de esta sucesión.

En todas las pruebas por inducción que hemos hecho hasta el momento, al demostrar
que la (k + 1)-ésima afirmación es verdadera sólo hemos utilizado la validez de la k-ésima
afirmación; inclusive, en cada caso simplificamos la hipótesis de inducción de tal manera que
abarcara sólo la afirmación anterior a la que querı́amos probar y no todas las anteriores. En
los ejemplos que trataremos a continuación sı́ necesitaremos hacer la hipótesis de inducción
como la anunciamos al principio de esta sección. La diferencia entre los casos que siguen y
los anteriores es que cada afirmación está ligada no sólo a la que la precede sino a una o
más de las anteriores. La práctica nos dirá cómo reconocer en qué caso nos encontramos;
mientras tanto, podemos siempre hacer la hipótesis de inducción en su forma más general
y, una vez que estemos en el tercer paso de la demostración inductiva, utilizar sólo lo que
necesitemos de la hipótesis.

Considerando que llegado este punto ya debe ser obvio para el lector el primer paso de la
inducción (es decir, identificar la sucesión de afirmaciones que abarca la afirmación general
que se quiere probar), de aquı́ en adelante ya no incluiremos éste en nuestras demostraciones.

4.11 Ejercicio. Probar por inducción que para n natural se tiene la fórmula:
n(n + 1)(2n + 1)
12 + 22 + 32 + · · · + n2 = .
6

4.12 Ejercicio. Probar la fórmula del ejercicio anterior en forma no inductiva como
sigue: Para k = 1, 2, . . . sea ak = (k + 1)3 − k 3 . Calcular la suma a1 + a2 + · · · + an de dos
maneras distintas e igualar. Despejar 12 + 22 + 32 + · · · + n2 .

4.13 Ejercicio. (∗) Calcular directamente la suma 13 +23 +33 +· · ·+n3 para n = 1, 2, 3
y 4; usar esto para proponer una fórmula para calcular la suma para cualquier natural n, y
probar la fórmula por inducción.

4.14 Ejercicio. Calcular la suma 13 + 23 + 33 + · · · + n3 utilizando un razonamiento


parecido al del ejercicio 4.12. (Sugerencia: En este caso deberá considerarse ak = (k +1)4 −k 4
para k = 1, 2, . . ..)

22
4.15 Ejercicio. (∗) Calcular la suma

1 × 1000 + 2 × 999 + 3 × 998 + · · · + 999 × 2 + 1000 × 1.

4.16 Ejercicio. (∗) Calcular la suma


1 1 1 1
+ + + ··· + .
1×3 3×5 5×7 999 × 1001

4.17 Ejercicio. (∗) Sea a0 ,a1 ,a2 ,. . . la sucesión de números definida recursivamente
como sigue: a0 = −1, a1 = 1 y, para n ≥ 2, an = an−1 +a 2
n−2
. Probar por inducción que para
n ≥ 1, an ≥ 0.

Haremos a continuación algunos ejemplos en que la demostración por inducción tiene


algunos puntos más delicados.

4.18 Ejemplo. La sucesión de Fibonacci f1 , f2 , f3 , . . . se define como sigue: f1 = 1,


f2 = 1 y, para n ≥ 3, fn = fn−1 + fn−2 . Construir los primeros 10 términos de la sucesión y
probar la siguiente fórmula que nos proporciona una definición no recursiva de la sucesión:
 √ n  √ n
1+ 5
2
− 1−2 5
fn = √ .
5
Solución. Construyamos los primeros 10 términos de la sucesión siguiendo la definición:
Los primeros dos términos son ambos 1 y, para construir cada uno de los términos siguientes,
sumemos cada vez los últimos dos que ya tengamos: 1,1,2,3,5,8,13,21,34, 55. Como pudimos
observar en la misma definición, para conocer un término es necesario conocer no sólo el
inmediato anterior sino los dos que le preceden. Es natural entonces pensar que una demos-
tración por inducción de una afirmación sobre todos los términos de la sucesión de Fibonacci
deba tener una hipótesis de inducción que abarque las afirmaciones correspondientes a los
dos términos que preceden al que se considera en ese momento. Por otro lado, los primeros
dos términos están dados de manera independiente y los demás se basan en ellos; por esta
razón, la base de inducción debe constar de la prueba de las dos afirmaciones correspon-
dientes a estos términos. Tomemos el lado derecho de la fórmula que queremos probar para
n = 1:  √ 1  √ 1
1+ 5
− 1−2 5 √ √ √
2 (1 + 5) − (1 − 5) 2 5
√ = √ = √ = 1 = f1 .
5 2 5 2 5
Hagamos ahora lo mismo para n = 2:
 √ 2  √ 2
1+ 5
− 1−2 5 √ √ √
2 (1 + 2 5 + 5) − (1 − 2 5 + 5) 4 5
√ = √ = √ = 1,
5 4 5 4 5

23
que es igual a f2 . Con esto concluimos la base de inducción. Ahora tomemos k ≥ 3 y hagamos
la hipótesis de inducción: “La fórmula es verdadera para todos los naturales menores que k”.
Tenemos entonces

fk = fk−1 + fk−2 (por definición)


 √ k−1  √ k−1  √ k−2  √ k−2
1+ 5 1− 5 1+ 5 1− 5
2
− 2 2
− 2
= √ + √ (por HI)
5 5
 √ k−2  √
  √ k−2  √ 
1+ 5 1+ 5
2
+ 1 − 1−2 5
2
1− 5
2
+ 1
= √
5
 √ k−2  √   √ k−2  √ 
1+ 5 3+ 5
2 2
− 1−2 5 3− 5
2
= √
5
Por otro lado, consideremos el miembro derecho de la fórmula para n = k:
 √ k  √ k  √ k−2  √ 2  √ k−2  √ 2
1+ 5
2
− 1−2 5 1+ 5
2
1+ 5
2
− 1−2 5 1− 5
2
√ = √
5 5
 √ k−2  √   √ k−2  √ 
1+ 5 1+2 5+5
2 4
− 1−2 5 1−2 5+5
4
= √
5
 √ k−2  √   √ k−2  √ 
1+ 5 3+ 5
2 2
− 1−2 5 3− 5
2
= √ .
5
Hemos obtenido lo mismo que tenı́amos arriba, ası́ que la fórmula también es verdadera para
n = k, y esto concluye la demostración. ♦

4.19 Ejemplo. Definamos una sucesión a0 ,a1 ,a2 ,. . . como sigue: a0 = 1 y, para n ≥ 1,
       
n n n n
an = a0 + a1 + a2 + · · · + an−1 .
0 1 2 n−1

Probar que todos los términos de la sucesión son impares.


Solución. Antes de empezar la demostración de que todos los términos son impares no-
temos primero que en la misma definición de la sucesión se hizo una recursión que utiliza no
sólo el término anterior al que se está definiendo sino todos los anteriores. Es natural entonces
pensar que para probar que un término an (n ≥ 1) es impar, debemos utilizar el que todos
los anteriores (a0 ,a1 ,a2 ,. . .,an−1 ) lo son; ası́ que en este caso, la hipótesis de inducción deberá
abarcar todos éstos. Conviene también escribir los primeros términos de la sucesión, pues el
análisis cuidadoso de varios términos en particular muchas veces da una idea de cómo hacer
la demostración general.

24
Tenemos:

a0 = 1,
 
1
a1 = ×1
0
= 1 × 1 = 1,
   
2 2
a2 = ×1+ ×1
0 1
= 1 × 1 + 2 × 1 = 3,
     
3 3 3
a3 = ×1+ ×1+ ×3
0 1 2
= 1 × 1 + 3 × 1 + 3 × 3 = 13,
       
4 4 4 4
a4 = ×1+ ×1+ ×3+ × 13
0 1 2 3
= 1 × 1 + 4 × 1 + 6 × 3 + 4 × 13 = 75,
         
5 5 5 5 5
a5 = ×1+ ×1+ ×3+ × 13 + × 75
0 1 2 3 4
= 1 × 1 + 5 × 1 + 10 × 3 + 10 × 13 + 5 × 75 = 541.

Observamos aquı́ que los coeficientes que van apareciendo son los del triángulo de
 Pascal, el
cual sabemos que es simétrico respecto a la vertical central (esto es, nr = n−r n

). También
sabemos que los términos centrales en los renglones pares (es decir, los de la forma nn ) son

2
todos números pares (pues son la suma de los dos números iguales arriba de él). Hechas estas
observaciones procedamos con la demostración.
La base de inducción es la prueba de que el primer término de la sucesión (es decir, a0 )
es impar, lo cual es cierto por definición. Tomemos k ≥ 1 y supongamos que a0 ,a1 ,a2 ,. . .,ak−1
son impares (ésta es nuestra hipótesis de inducción). Probaremos que ak es impar. Dividimos
la prueba en dos casos: cuando k es impar y cuando k es par. En el primer caso, factorizando
los coeficientes binomiales con sus simétricos, tenemos
     
k k k 
ak = 1 + (a1 + ak−1 ) + (a2 + ak−2 ) + · · · + k−1 a k−1 + a k+1 .
1 2 2
2 2

Ahora utilizamos la hipótesis de inducción: como cada ai (con 1 ≤ i ≤ k − 1) es impar, cada


una de las sumas a1 + ak−1 , a2 + ak−2 , . . ., a k−1 + a k+1 es un número par; con esto ya es claro
2 2
que ak es impar, y aquı́ termina la prueba para el caso en que k sea impar. En el caso en
que k sea par, agrupamos de la misma manera pero nos sobrará un término sin agrupar:
     
k k k
ak = 1 + (a1 + ak−1 ) + (a2 + ak−2 ) + · · · + k a k .
1 2 2
2

25
Sin embargo, el término no agrupado también es par pues el coeficiente binomial en él lo es.
Esto concluye la prueba en el caso en que k es par. Hemos completado satisfactoriamente
los pasos de la inducción en todos los casos. ♦

4.20 Proposición. Todo conjunto con n elementos tiene 2n subconjuntos


Demostración. El resultado es obvio para cuando n = 0 pues el conjunto con 0 elementos
sólo tiene un subconjunto que es él mismo. Sea n ≥ 1; HI: “Todo conjunto con n−1 elementos
tiene 2n−1 subconjuntos.” Consideremos el conjunto X = {x1 , x2 , . . . , xn } con n elementos.
Queremos utilizar la HI para probar que X tiene 2n subconjuntos. Consideremos el conjunto
Y obtenido al quitarle a X el elemento xn . Por HI, Y tiene 2n−1 subconjuntos. Ahora bien,
los subconjuntos de X podemos dividirlos en dos clases: los que no tienen al elemento xn
(es decir, los que están contenidos en Y ) y los que sı́ lo tienen. El número de conjuntos de
las dos clases es el mismo pues cada conjunto de la segunda clase se obtiene adjuntando el
elemento xn a uno de los conjuntos de la primera. Entonces, por HI, cada una de estas clases
tiene 2n−1 conjuntos; en total X tendrá 2n−1 + 2n−1 = 2 × 2n−1 = 2n subconjuntos, como
querı́amos probar. Esto termina la demostración. ♦

4.21 Ejercicio. Sea X = {x1 , x2 , x3 , x4 }. Encontrar las dos clases de subconjuntos de


X de que se habla en la demostración de 4.20 y aparear los conjuntos de una clase con los
de la otra como indica esa prueba.

4.22 Ejercicio. (∗) Demostrar que todo conjunto tiene la misma cantidad de subcon-
juntos con un número par de elementos que con un número impar.

4.23 Ejercicio. (∗) Si a1 = 1 y, para n ≥ 2, an = n + (−1)n an−1 , ¿cuánto valen a1000 ,


a2001 , a3002 , a4003 ?

4.24 Ejercicio. (∗) Probar por inducción la fórmula siguiente para n natural:
     
n n n
+ + ··· + = 2n .
0 1 n

(Sugerencia: Utilizar la Fórmula de Pascal.)

4.25 Ejercicio. (∗) Probar por inducción la fórmula siguiente para n y k enteros con
0 ≤ k ≤ n:          
k k+1 k+2 n n+1
+ + + ··· + = .
k k k k k+1
(Sugerencia: Usar la Fórmula de Pascal.)

4.26 Ejercicio. (∗) La siguiente afirmación es obviamente falsa: “En cualquier lista
de n números, todos son iguales entre sı́.” Determinar cuál es el error en la “demostración”

26
por inducción que presentamos a continuación (es decir, encontrar en qué momento el pro-
cedimiento que se sigue en la supuesta demostración es incompleto o incorrecto): “BI: Para
n = 1 la afirmación es verdadera pues sólo hay un número en la lista. HI: Supongamos que el
resultado es cierto para cualquier lista de n números y tomemos una lista de n + 1 números:
a1 , a2 , . . . , an , an+1 . Entonces, por HI, los primeros n números a1 , a2 . . . , an son iguales entre
sı́; aplicando también la hipótesis de inducción a los últimos n números: a2 , . . . , an , an+1 ,
éstos son también iguales entre sı́. Pero entonces todos son iguales a a2 , ası́ que todos son
iguales entre sı́.”

4.27 Ejercicio. (∗) La siguiente afirmación es obviamente falsa: “Para construir una
red carretera en un paı́s de manera que cualesquiera dos ciudades estén conectadas mediante
la red, es suficiente que a cada ciudad llegue al menos una carretera (entendiendo que las
carreteras sean todas de doble sentido)”. Determinar cuál es el error en la “prueba” por
inducción que presentamos a continuación: “Para dos ciudades el resultado es claramente
cierto. Supongamos por inducción que un paı́s con n ciudades cumple la propiedad y agre-
guemos una ciudad C; por hipótesis, de C sale una carretera, digamos a D. Entonces, a través
de D, la ciudad C está conectada con las demás, y ası́, todas las ciudades están conectadas
entre sı́.”

4.28 Ejercicio. (∗) La siguiente afirmación es obviamente falsa: “El conjunto N de los
números naturales es finito.” Determinar cuál es el error en la “prueba” por inducción que
presentamos a continuación: “Para cada natural n sea An = {n}. Sabemos que la unión de
todos los conjuntos An nos da el conjunto N, y que la unión de dos conjuntos finitos es finito.
Entonces BI: A1 ∪ A2 es finito. HI: Supongamos que A1 ∪ A2 ∪ · · · ∪ An−1 es finito para
cierta n ≥ 3. Entonces, A1 ∪ A2 ∪ · · · ∪ An = (A1 ∪ A2 ∪ · · · ∪ An−1 ) ∪ An , que es la unión
de dos conjuntos finitos (usando la HI), ası́ que A1 ∪ A2 ∪ · · · ∪ An también es finito. En
conclusión, N es finito.”

27
5. Caminos

Analizaremos en esta sección un ejemplo básico de cuenta de caminos siguiendo lı́neas en


figuras. En este ejemplo aprenderemos dos técnicas para la resolución de problemas de este
estilo y tendremos oportunidad de practicar algunas técnicas de combinatoria aprendidas
antes.

5.1 Ejemplo. En una cuadrı́cula de m por n, sea A el vértice inferior izquierdo y sea
B el vértice superior derecho. ¿Cuá ntos caminos hay de A a B siguiendo las lı́neas de la
figura, si sólo se puede avanzar hacia la derecha y hacia arriba?
Solución. Ilustremos un camino con m = 4 y n = 6.

B
......................................
........
......
......
......
............................................
.....
.....
......
......
.....
............................................................................................
......
......
......
......
......
......
......
......
......
......
........................................
A
Primera forma. Notemos que cada camino debe recorrer una distancia total de m + n =
10 segmentos, de los cuales 6 segmentos se recorren horizontalmente y 4 verticalmente.
Entonces cada camino se puede identificar con una “palabra” de 10 letras que usa 6 letras
H (horizontal) y 4 letras V (vertical). (Por ejemplo, el camino de la figura está representado
por la palabra HV V HHHV HV H.) Como sabemos, el número de estas palabras es 10

4
.

Segunda forma. Observemos que hay dos maneras para llegar a cada vértice que no forme
parte del lado izquierdo o del lado de abajo de la cuadrı́cula (a los cuales sólo hay una forma
de llegar): verticalmente (desde el vértice inmediatamente abajo de él) y horizontalmente
(desde el vértice inmediatamente a la izquierda de él). Entonces, el número de caminos que
llegan a uno de esos vértices es la suma de los caminos que llegan a los vértices adyacentes
a la izquierda y abajo de él. Ası́ podemos poner en la figura, junto a cada vértice, el número
de caminos que llegan a él:

28
1 5 15 35 70 126 210
B
1 4 10 20 35 56 84
1 3 6 10 15 21 28
1 2 3 4 5 6 7
1 1 1 1 1 1 1
A

5.2 Ejercicio. (∗) Dentro de un cubo de alambre C de dimensiones 5 × 5 × 5 se colocan


alambres dividiendo C en cubos de dimensiones 1 × 1 × 1. Llámese A al vértice inferior
izquierdo de la cara anterior de C y sea B el vértice opuesto a A en C (es decir, B es el
vértice superior derecho de la cara posterior de C). ¿Cuántos “caminos” diferentes llegan del
punto A al punto B siguiendo los alambres del cubo, si las únicas direcciones permitidas son:
hacia atrás, hacia la derecha y hacia arriba?

5.3 Ejercicio. (∗) Usar caminos para probar la fórmula de naturales n:


 2  2  2  2  
n n n n 2n
+ + + ··· + = .
0 1 2 n n

(Sugerencia: Contar caminos en una cuadrı́cula de n × n como en la primera forma en el


ejemplo 5.1, fijándose en los puntos donde el camino atraviesa la diagonal que va de la esquina
superior izquierda a la esquina inferior derecha.)

5.4 Ejercicio. Usar caminos para probar la fórmula de Pascal


     
n+1 n n
= + ,
r+1 r r+1

para r y n números enteros con 0 ≤ r < n. (Sugerencia: Comparar las dos formas de resolver
el ejemplo 5.1.)

5.5 Ejercicio. (∗) Siguiendo las lı́neas de la figura, ¿cuántos caminos hay para ir del
punto A al punto B que no pasen dos veces por el mismo punto y que sólo avancen hacia
abajo y hacia los lados pero no hacia arriba? [3er Examen Nacional]

29
A ..
......
... .....
... ...
.... ...
.
...................................
... ..... .
. .. .. ..... .....
.
.. ... .. ...
... . .... . ...
.....................................................................
... ... . .. . .
.... ..... .... .... .... .....
.. ... ... ... ... ...
... .. . .. . .
.................................................................................................
...... .. .... .. ..... ..
... .....
. .. .. .... .....
..
. . . .... .... .... .... ... ...
... ... .. ... ... ... ... ...
. . .. . . . .
................................................................................................................................
.........
. ... ... ... ... ... ... ... ...
.
.... ..... .... ..... .... ..... .... .... .... .....
. ... ... ... ... ... ... ... ... ...
... .. . .. .. .
.................................................................................................................................................................
... ... .. .. .. .. .. . .. . .. ..
... .... .... ..... .... ..... .... ..... .... ..... .... .....
.
..
. ... ... .. ... .. ... ... ... ... ... ...
. ...... ...... ...... ...... ..... ...
... .. .. .. .. ..
..................................................................................................................................................................................................
... ....
. . ... . ... .. ... .. ... .. .. .. .
.
...
... ..
. .
.. ... ..
. .
. . ... ... . ... ... . ... ... .... ... .....
. .
. ... ... ... ... ... .. . ... .. . ... .. . ... .. . ...
..
....................................................................................................................................................................................................
B

5.6 Ejercicio. (∗) ¿Cuántos caminos hay del punto A al punto B siguiendo las lı́neas
de la figura si las direcciones permitidas son →, %, &, ., - (es decir, cualquier sentido está
permitido salvo ←) y no se permite pasar dos veces por el mismo punto?
........................... ........................... ........................... ...........................
... ...
..
.. ... ..
..
... ..
..
...
.. ...
..........................
...
..........................
...
..........................
...
...
. ....
.. ...
.. ...
.. ..... . ... . ... . ...
......................
. ...
... ... ..
. . ..
. . ..
. . .. ...
.
.. ...
. ... ...
. ... ...
. ... ...
. .... ...
.. ... .. ... .. ... .. ... .. ...
A• ..
...
...
...
... .. ....
.
.. ..
......................
...
.... ..
.
. ..
.
.. ..
......................
...
.... ..
.
...
.
.. ..
......................
...
.... ..
.
...
.
.. ..
......................
...
....
•B
.
..
..
...
.. .
........................... .
........................ .. .
........................ .. ......................... .
.
........................ . ... . . . . .
... ... ...
... ... ... ... ... ... ... ...
... ... ... ... ... ... ... ...
.......................... ...
.....................
.. .......................... ..........................

30
6. Inclusión y Exclusión

Empecemos esta sección analizando un ejemplo que nos dará la clave del llamado Prin-
cipio de Inclusión y Exclusión.

6.1 Ejemplo. ¿Cuántos números menores que 10 000 no son divisibles ni por 2, ni por
3, ni por 5?
Solución. A 10 000 habrá que restarle la cantidad de números divisibles por alguno de 2,
3 o 5. Sin embargo esto hay que hacerlo con cuidado para evitar repeticiones; por ejemplo,
los números que son divisibles tanto por 2 como por 3 se consideran dos veces: al contar los
divisibles por 2 y al contar los divisibles por 3. Vamos a determinar primero, por separado,
cuántos múltiplos hay de cada una de las distintas combinaciones entre 2, 3 y 5. Hay:

5 000 números divisibles por 2,


3 333 divisibles por 3,
2 000 divisibles por 5,
1 666 divisibles por 6,
1 000 divisibles por 10,
666 divisibles por 15 y
333 divisibles por 30.

Al restarle a 10 000 la cantidad de números divisibles por 2 y luego los divisibles por 3 y a
continuación los divisibles por 5:

10 000 − (5 000 + 3 333 + 2 000),

los que son divisibles por 6, por 10 o por 15 pero no por 30 se habrán quitado dos veces
cada uno, y los que son múltiplos de 30 se habrán quitado tres veces. Entonces al agregar a
la cuenta los que son múltiplos de 6, de 10 o de 15, los que son divisibles por 30 se habrán
quitado primero tres veces al restar los múltiplos de 2, de 3, y de 5, y después se habrán
vuelto a sumar tres veces al sumar los múltiplos de 6 y los de 10 y los de 15, ası́ que tendremos
que restarlos. La respuesta al ejemplo es pues:

10 000 − (5 000 + 3 333 + 2 000) + (1 666 + 1 000 + 666) − 333 = 2 666.♦

6.2 Ejercicio. En cierta escuela hay 100 alumnos. De ellos 50 saben inglés, 30 saben
alemán y 30 saben francés. Además 10 saben inglés y francés, 14 saben francés y alemán,
11 saben inglés y alemán, y 6 saben los tres idiomas. Determinar cuántos alumnos no saben
ninguno de los tres idiomas.

El procedimiento que utilizamos en el ejemplo anterior puede generalizarse, según veremos


en el siguiente principio.

31
6.3 Proposición. Principio de Inclusión y Exclusión. Supongamos que tenemos n
conjuntos A1 , A2 , . . . , An (posiblemente con elementos en común). Entonces el número total
k de elementos que tienen entre todos es igual a k1 − k2 + k3 − k4 + − · · · kn , donde k1 es la
suma de los elementos que pertenecen a (por lo menos) uno de los conjuntos, k2 es la suma
de los elementos que pertenecen a (por lo menos) dos de los conjuntos, y ası́ sucesivamente
hasta kn , que es el número de elementos en común a todos los conjuntos. (Utilizando el
lenguaje usual de teorı́a de conjuntos donde |X| denota el número de elementos de un
conjunto X, ∪ es el sı́mbolo usual de unión y ∩ es el sı́mbolo usual de intersección, tenemos:
k = |A1 ∪ A2 ∪ · · · ∪ An |, k1 = |A1 | + |A2 | + · · · + |An |, k2 = |A1 ∩ A2 | + |A1 ∩ A3 | + · · · + |A1 ∩
An | + |A2 ∩ A3 | + · · · + |An−1 ∩ An |, y ası́ sucesivamente hasta kn = |A1 ∩ A2 ∩ · · · ∩ An |.)
Demostración. Tomemos un elemento cualquiera y supongamos, por ejemplo, que el ele-
mento pertenece a los conjuntos Ai1 , Ai2 , . . . , Air para cierta r, y sólo a éstos. Entonces el
número de veces que dicho elemento se considera en la suma k1 − k2 + k3 − k4 + − · · · kn es
         
r r r r r
− + − + −··· ,
1 2 3 4 r
que, por el ejercicio 4.22, es igual a 0r , que es 1. Entonces la suma k1 − k2 + k3 − k4 + − · · · kn


cuenta cada elemento exactamente una vez, que es lo que querı́amos demostrar. ♦

Para el siguiente ejemplo recordemos que todo número natural N se puede escribir en
forma única como producto de potencias de primos p1 < p2 < · · · < pr : N = pa11 pa22 · · · par r
(a1 , a2 , . . . , ar son naturales) y que a ésta se le llama la descomposición canónica del
número N . Recordemos también que se dice que dos números son primos relativos cuando
su máximo común divisor es 1 (en otras palabras, cuando los primos que aparecen en sus
descomposiciones canónicas, son distintos).

6.4 Ejemplo. Sea N = pa11 pa22 · · · par r la descomposición canónica del número natural
N . Se llama ϕ(N ), o función ϕ de Euler al número de enteros menores que N y primos
relativos con N . (Por ejemplo ϕ(12) = 4 pues los enteros menores que 12 y primos relativos
con 12 son 4, a saber, 1, 5, 7 y 11.) Probar que
ϕ(N ) = pa11 −1 pa22 −1 · · · par r −1 (p1 − 1)(p2 − 1) · · · (pr − 1).
Solución. Aplicamos el Principio de Inclusión y Exclusión. Los múltiplos de cada pi son
N
pi
los múltiplos de cada pi pj (para i 6= j) son pN
; i pj
; los múltiplos de los productos pi pj pk
(para i, j y k ı́ndices distintos) son pi pNj pk , y ası́ sucesivamente. Entonces ϕ(N ) es igual a:

N − ( pN1 + · · · + N
pr
) + ( p1Np2 + · · · + N
pr−1 pr
) − · · · + (−1)r p1 pN
2 ···pr
=

pa11 pa22 · · · par r − (pa11 −1 pa22 · · · par r + pa11 pa22 −1 · · · par r + · · · + pa11 pa22 · · · par r −1 )+

a −1
(pa11 −1 pa22 −1 · · · par r + · · · + pa11 · · · pr−1
r−1
par r −1 ) + · · · +

(−1)r pa11 −1 pa22 −1 · · · par r −1 .

32
Podemos observar que los sumandos en la expresión anterior son todas las expresiones de la
forma pb11 pb22 · · · pbrr , donde cada bi es igual a ai o a ai − 1; además la paridad del número de
sumandos que son ai − 1 determina el que se sume o se reste el término correspondiente en
la expresión. Entonces la expresión se puede simplificar:

ϕ(N ) = (pa11 − pa11 −1 )(pa22 − pa22 −1 ) · · · (par r − par r −1 )


= pa11 −1 (p1 − 1)pa22 −1 (p2 − 1) · · · par r −1 (pr − 1),

que es la fórmula que querı́amos probar. ♦

6.5 Ejercicio. Usar la fórmula probada en el ejemplo anterior para calcular ϕ(600),
ϕ(1995), ϕ(23) y ϕ(128).

6.6 Ejercicio. (∗) Se dice que una mano de dominó tiene falla si alguno de los números
entre el 0 y el 6 no aparece en la mano (cada número faltante es una falla); por ejemplo la
mano

2 1 , 5 5 , 3 1 , , 1 , 6 5 , 2

tiene falla a 40 s. Probar que el número de manos de dominó que no tienen falla es 501 015.

6.7 Ejercicio. (∗) En una oficina hay 10 empleados. Cada uno es especialista en una
labor distinta a la de los demás. Para no aburrirse, les gusta intercambiar sus puestos; sin
embargo, el buen funcionamiento de la oficina exige que en cada momento haya exactamente
4 empleados trabajando en su especialidad. ¿Cuántas distribuciones de los puestos se pueden
hacer bajo estas condiciones?

33
7. Probabilidad Combinatoria

Intuitivamente, la probabilidad calcula la proporción de casos en los que cierto experi-


mento ocurre en relación con el total de resultados posibles.

El conjunto de todos los resultados posibles de un experimento se llama espacio mues-


tral. Cualquier subconjunto del espacio muestral se llama suceso.

Analicemos algunos ejemplos en los que el espacio muestral Ω es finito y, en ese caso,
definamos la probabilidad de que ocurra un suceso S, en sı́mbolos P (S), como
|S|
P (S) = .
|Ω|
(Aquı́ se está suponiendo que todos los resultados del experimento tienen la misma proba-
bilidad de ocurrir; más adelante se verán ejemplos de distinta ı́ndole.)

Se piensa entonces que la probabilidad es una función que va del conjunto de sucesos al
conjunto de los números racionales. El valor de un suceso es 0 cuando no puede ocurrir y es
1 cuando es seguro que ocurre.

En muchos de nuestros problemas aparece el conunto de los números naturales del 1 al


n. Para simplificar, denotaremos a este conjunto por n, es decir,

[n] = {1, 2, . . . , n}.

7.1 Ejemplo. El experimento consiste en lanzar un dado y observar el número que


queda arriba. Calcular la probabilidad de que el número que quede arriba sea el 1 y también
calcular la probabilidad de que el número que quede arriba sea par.
Solución. Aquı́ M = [6]. En el primer caso el suceso es S = {1} y P (S) = 61 . En el
segundo caso el suceso es T = {2, 4, 6} y P (T ) = 36 = 12 . ♦

7.2 Ejemplo. El experimento es lanzar una moneda 2 veces y observar la sucesión de


águilas a y soles s que se obtiene. Determinar la probabilidad de que se observen dos águilas.
Solución. El espacio muestral puede ser

Ω = {aa, as, sa, ss}

y entonces el suceso es S = {aa} y P (S) = 41 . ♦

7.3 Ejemplo. Calcular la probabilidad de que al lanzar una moneda 3 veces se mues-
tren al menos dos águilas.
Solución. Aquı́ podemos definir

Ω = {aaa, aas, asa, saa, ass, sas, ssa, sss}.

34
El suceso es S = {aas, asa, saa, aaa} y entonces la probabilidad buscada es P (S) = { 48 =
1
2
.♦

7.4 Ejemplo. Determinar la probabilidad de que al lanzar dos dados lo que sumen las
caras que se ven arriba sea 6.
Solución. Conviene definir Ω = [6] × [6] y entonces
S = {(1, 5), (2, 4), (3, 3), (4, 2), (5, 1)},
5
de donde la probabilidad es 36
≈ 0.14. ♦

Veamos algunas propiedades que ya hemos podido observar en los ejemplos y hagamos
algunos comentarios sobre ellas.

De aquı́ en adelante Ω denota al espacio muestral en cuestión.

7.5 Nota. La probabilidad de que algo ocurra es un número entre 0 y 1. (Esto es obvio
pues, como el suceso S es subconjunto del espacio muestral Ω, entonces |S| ≤ |M |.) Es 0
cuando es imposible que ocurra (es decir, P (∅) = 0), y es 1 cuando es seguro que debe ocurrir
(o sea, P (M ) = 1). En el caso en que Ω es conjunto finito, para todo S ⊂ Ω se tiene que
P (S) ∈ Q.

7.6 Nota. Si dos cosas no pueden ocurrir simultáneamente, la probabilidad de que


ocurra una o la otra (es decir, cualquiera de las dos) es la suma de las probabilidades. En otras
palabras, si S y T son sucesos ajenos (es decir, S ∩T = ∅), entonces P (S ∪T ) = P (S)+P (T ).
(Esto es claro pues |S ∪ T | = |S| + |T |.)

Retomemos aquı́ el ejemplo arriba en el que se pide calcular la probabilidad de que al


lanzar tres monedas al aire salgan al menos dos águilas. Arriba calculamos la probabilidad
de S ∪ T donde S = {ass, sas, ssa, } y T = {aaa}, pero podrı́amos haber calculado por
separado las probabilidades P (S) = 83 y P (T ) = 81 .

Observemos que la propiedad anterior no serı́a válida si no pidiéramos que los suce-
sos fueran mutuamente excluyentes, es decir, si hubiera la posibilidad de que ocurrieran
simultáneamente; por ejemplo, la probabilidad de que al lanzar un dado lo que salga sea un
número mayor que 3 o que sea un número par es 46 (el suceso es {2, 4, 5, 6}) y no 63 + 36 = 1,
que serı́a la suma de las probabilidades de los sucesos S = {4, 5, 6} y T = {2, 4, 6} (los casos
4 y 6 son comunes a los dos y se estarı́an contando dos veces al sumar las probabilidades).

7.7 Nota. La probabilidad de que ocurran dos cosas en un orden determinado es el


producto de las probabilidades. En este caso estamos diciendo que si S1 es un suceso en un
espacio muestral Ω1 y S2 es un suceso en un espacio muestral Ω2 , entonces P (S1 × S2 ) =
P (S1 )P (S2 ), lo cual es claro pues dados dos conjuntos A y B, el número de elementos del
producto cartesiano A × B es |A||B|.

35
Retomemos el ejemplo arriba en el que querı́amos calcular la probabilidad de que al
lanzar dos monedas al aire el resultado en ambas sea águila. En lugar de lo hecho arriba,
podrı́amos haber definido Ω = {a, s} y S = {a} y calcular P (S)P (S) = 21 12 = 14 .

Dado S ⊂ Ω denotamos por S al complemento de S, es decir, al conjunto Ω \ S = {x ∈


Ω:x∈/ S}

7.8 Corolario. Si la probabilidad de que algo ocurra es p, entonces la probabilidad de


que no ocurra es 1 − p.
Demostración. Esto es claro por la propiedad (2) pues para S ⊂ Ω, S y S son conjuntos
ajenos cuya unión es Ω, ası́ que 1 = P (M ) = P (S) + P (S). ♦

Veamos más ejemplos en los que podremos observar que hay que escoger con cuidado el
espacio muestral para que represente verdaderamente el problema que se quiere resolver.

7.9 Ejemplo. El experimento es sacar 2 pelotas de una caja en la que hay 2 pelotas
rojas y 3 azules. Se quiere calcular la probabilidad de que las dos pelotas escogidas tengan
distinto color y compararla con la probabilidad de que tengan el mismo color.
Solución. Para definir el espacio muestral conviene numerar las pelotas y pensar que las
rojas son la 1 y la 2, y que de la 3 a la 5 son azules; entonces el espacio muestral es

Ω = {{1, 2}, {1, 3}, {1, 4}, {1, 5}, {2, 3}, {2, 4}, {2, 5}, {3, 4}, {3, 5}, {4, 5}}

y S = {{1, 3}, {1, 4}, {1, 5}, {2, 3}, {2, 4}, {2, 5}},
6
ası́ que P (S) = 10 = 53 = 0.6. La probabilidad de que las dos pelotas tengan el mismo color
4
se calcula considerando el suceso T = {{1, 2}, {3, 4}, {3, 5}, {4, 5}}, y aquı́ P (T ) = 10 =
0.4 < P (S) (o, de otra manera, como T = S, P (T ) = 1 − P (S) = 1 − 53 = 25 = 0.4). ♦

7.10 Ejemplo. Como en el ejemplo anterior, se tiene una caja en la que hay 2 pelotas
rojas y 3 azules, pero ahora el experimento consiste en sacar una pelota, observar su color,
volverla a meter, y sacar otra vez una pelota. Calcular la probabilidad de que las dos pelotas
escogidas tengan distinto color.
Solución. Numeremos las pelotas como en el ejemplo anterior. En este caso

Ω = [5] × [5] y

S = {(1, 3), (3, 1), (1, 4), (4, 1), (1, 5), (5, 1), (2, 3), (3, 2), (2, 4), (4, 2), (2, 5), (5, 2)},
por lo que P (S) = 12
25
= 0.48, que es menor que el resultado del ejemplo anterior, lo cual
resultaba intuitivamente obvio. ♦

7.11 Ejemplo. Dentro de cierto grupo de 4 caballos numerados del #1 al #4 se ha


observado que la frecuencia con que el caballo #1 gana es el doble que con la que gana el
#2; que éste a su vez gana el doble de veces que el #3, y que el #3 gana el doble de veces

36
que el #4. Encontrar la probabilidad de que en la próxima carrera el caballo ganador sea el
#3.
Solución. Tenemos que representar en el espacio muestral las condiciones de que unos
ganan el doble de veces que otros. Podemos entonces asignar al caballo 4 el número 1, al
caballo 3 los números 2 y 3, al caballo 2 los números 4, 5, 6 y 7, y al caballo 1 los números
2
del 8 al 15. De esta manera Ω = [15], S = {2, 3} y la probabilidad es 15 ≈ 0.13. ♦

Para eliminar complicaciones técnicas, en los dos ejemplos siguientes consideraremos


el año con 365 dı́as (sin contar en ningún caso el 29 de febrero) y supondremos que la
distribución de los cumpleaños es pareja a lo largo del año.

7.12 Ejemplo. Encontrar la probabilidad de que una persona determinada haya nacido
en enero o febrero.
59
Solución. Ω = [365], S = [59] y P (S) = 365 ≈ 16 . ♦

7.13 Ejemplo. Encontrar la probabilidad de que en un grupo de 59 personas al menos


2 tengan el mismo cumpleaños.
Solución. Notemos que este ejemplo difiere del anterior en que las fechas de cumpleaños
no se comparan con fechas fijas sino entre sı́. Veremos que los resultados son muy distintos.
Para resolver el ejemplo resulta más fácil contar la probabilidad opuesta: que no haya ningún
cumpleaños repetido. Utilizaremos repetidamente la propiedad (3). Consideremos un orden
fijo para las personas. La probabilidad de que el segundo cumpleaños sea distinto del primero
es 364
365
. La probabilidad de que el tercero sea distinto de los dos anteriores es 363 365
, y ası́
sucesivamente. El resultado es
364 × 363 × · · · × 307
1− ,
36558
que es aproximadamente igual a 0.995. Esto quiere decir que de 1000 grupos de 59 personas
cada uno, se espera que en sólo 5 de los grupos no haya cumpleaños comunes. (Compárese
este resultado con el del ejemplo anterior. Resulta que basta con 23 personas para que la
probabilidad de que haya cumpleaños repetidos entre ellas sea mayor que 21 .) ♦

7.14 Ejemplo. Encontrar la probabilidad de que al lanzar una moneda al aire 10 veces
caigan exactamente 5 águilas.
Solución. Como antes, escribamos a por águila y s por sol. El espacio muestral Ω consta
de todas las sucesiones de longitud 10 formadas por a y s, de manera que |Ω| = 210 = 1024.
El suceso consta de los elementos de Ω que tienen exactamente 5 a0 s, ası́ que |S| es el número
de formas en que se pueden escoger 5 posiciones (donde aparezcan las a0 s) dentro de un total
de 10, es decir, 10
5
252
= 252. Entonces P (S) = 1024 ≈ 0.25. ♦

En forma análoga a la resolución del ejemplo anterior tenemos que la probabilidad de


que de un total de 20 lanzamientos de la moneda 10 salgan águila es 2120 20

10
, que es apro-
ximadamente igual a 0.176. Se puede demostrar que mientras más lanzamientos se hagan,

37
la probabilidad de que la mitad de las veces salga águila es menor. Esto no contradice la
afirmación de que si una moneda se lanza al aire un número grande de veces se espera que
un número cercano a la mitad de las ocasiones caiga águila; la explicación para esto es que la
idea de “cercanı́a” debe manejarse en forma relativa al tamaño del número; por ejemplo, en
el caso de 10 lanzamientos podrı́amos decir que los casos en que salieran entre 3 y 7 águilas
son todos “cercanos” a la mitad, y en el caso de 20 lanzamientos dirı́amos que los casos
“cercanos” a la mitad son entre 5 y 15.

7.15 Ejercicio. Encontrar la probabilidad de que al lanzar una moneda al aire 10 veces
salga águila entre 3 y 7 veces.

Como ya hemos visto, se pueden considerar distintos espacios muestrales para resolver un
determinado problema y, en cada caso, el suceso del cual se quiere calcular la probabilidad
es diferente, ası́ que los cálculos también lo son, aunque, claro, el resultado final debe ser el
mismo. En el siguiente ejemplo presentamos varias formas de resolver el problema según el
espacio muestral que se escoja.

7.16 Ejemplo. ¿Cuál es la probabilidad de que al escoger dos subconjuntos de 4


elementos dentro de un conjunto de 10 elementos, los subconjuntos tengan al menos un
elemento en común?
Solución. Es más fácil contar la probabilidad contraria, es decir, la probabilidad de que los
dos subconjuntos escogidos no tengan elementos en común. Consideremos distintos espacios
muestrales Ω y los respectivos sucesos S con complemento S:

Primera forma. Sea P4 = {A ⊂ [10] : |A| = 4}, es decir, P4 tiene por elementos a
los subconjuntos de [10] que tienen 4 elementos. Tomemos Ω = P4 × P4 . En este caso
10 6
 
|S| = 4 4 , ası́ que
6·5·4·3
10 6 6
  
4 4 4
4·3·2·1
10·9·8·74·3·2·1 6·5·4·3 13
P (S) = 1 − P (S) = 1 − 10 2
=1− 10
 =1− 1− = .
= 10 · 9 · 8 · 7 14

4 4

Segunda forma. Sea P4 como arriba. Supongamos que un conjunto de 4 elementos ya está
escogido; entonces queremos calcular la probabilidad de que al escoger otro conjunto, éste
sea ajeno con el primero. En este caso tomemos Ω = P4 . Aquı́ |S| = 64 y |M | = 10 4
, ası́
que
6·5·4·3
6

4
4·3·2·1
10·9·8·74·3·2·1 6·5·4·3 13
P (S) = 1 − P (S) = 1 − 10 = 1 − 1− = .
4
= 10 · 9 · 8 · 7 14
Tercera forma. Como en la segunda forma, supongamos que un conjunto de 4 elementos
ya está escogido; entonces queremos calcular la probabilidad de que al escoger otro conjunto,
éste sea ajeno con el primero. Sea Ω = {(c1 , c2 , c3 , c4 ) ∈ [10] : ci 6= cj para i 6= j}. y
supongamos que el conjunto ya escogido tiene elementos a1 , a2 , a3 , a4 ; en este caso S =

38
{(c1 , c2 , c3 , c4 ) ∈ Ω : para cada i, j ci 6= aj }, |S| = 6 · 5 · 4 · 3 y |M | = 10 · 9 · 8 · 7 y, entonces,
6·5·4·3 13
P (S) = 1 − P (S) = 1 − = .♦
10 · 9 · 8 · 7 14

7.17 Ejemplo. Alejandra y Delia van a jugar un juego. Alejandra lanzará un dado y
le dará una moneda a Delia cada vez que lo que salga en el dado no sea 2. Si se quiere que
ninguna de las dos jugadoras tenga ventaja sobre la otra, ¿cuántas monedas deberá pagar
Delia cada vez que salga el 2?
Solución. Como la probabilidad de que salga el 2 es 61 , se espera que de cada 6 veces una
de ellas salga 2; entonces Delia deberá darle 5 monedas cuando esto ocurra. En 6 juegos se
espera que Alejandra pierda 5 veces una moneda y gane una vez 5 monedas, por lo que su
ganancia esperada es de 0. ♦

Generalicemos el ejemplo que acabamos de estudiar. Si algo puede ocurrir de un total


de r formas (mutuamente excluyentes) con probabilidades p1 , p2 , . . ., pr (de manera que
p1 + p2 + · · · + pr = 1) y ganancias respectivas g1 , g2 , . . ., gr , entonces el valor esperado E
del suceso se define como
E = g1 p1 + g2 p2 + · · · + gr pr .
Para entender mejor esta nueva definición analicemos en el ejemplo anterior cuál es la ga-
nancia esperada de Alejandra si Delia le da 5 monedas cada vez que salga el 2. Llamemos p1
a la probabilidad de que no salga el 2 y p2 a la probabilidad de que sı́ salga; entonces p1 = 65 ,
p2 = 16 , g1 = −1 (pues Alejandra pierde una moneda cuando no sale el 2) y g2 = 5. El valor
esperado del suceso (ganancia esperada para Alejandra) es E = (−1) × 56 + 5 × 61 = 0.

7.18 Ejemplo. En el juego de ruleta hay 36 números (del 1 al 36) y además los sı́mbolos
0 y 00, con los que el dueño de la ruleta gana automáticamente. Se ofrece pagar 36 veces lo
apostado cada vez que salga el número al que uno apostó (es decir, si uno indica uno de los
36 números y paga una ficha por jugar, en caso que al girar la ruleta la bolita se detenga en
el número escogido, el dueño de la ruleta le devolverá su ficha al jugador y le dará otras 35
más). ¿Cuál es la ganancia esperada de un jugador?
Solución. Llamemos p1 a la probabilidad de que el jugador gane, y p2 a la probabilidad
1
de que pierda. Entonces p1 = 38 , p2 = 37
38
1
, g1 = 35 y g2 = −1; ası́ E = 35 × 38 37
+ (−1) × 38 =
−2
38
, que es aproximadamente igual a −0.05. Esto quiere decir que el jugador espera perder
alrededor de un 5 % de lo apostado; en otras palabras, el dueño de la ruleta espera ganar el
5 % de lo que se apueste. ♦

7.19 Ejercicio. (∗) Cuatro equipos A, B, C, D entran a un torneo de basquetbol. Al


principio juegan A contra B, y C contra D; en cada juego se elimina al perdedor. Los dos
ganadores se enfrentan y el que gane ese juego se determina como ganador del torneo. Escribir
un espacio muestral apropiado y el suceso correspondiente para determinar la probabilidad
de que B sea el ganador.

39
7.20 Ejercicio. (∗) Un grupo de 3 mujeres y 3 hombres se dividirá en dos equipos con
3 miembros cada uno. Definir un espacio muestral y el suceso correspondiente que sirvan
para encontrar la probabilidad de que en uno de los equipos queden todos los hombres y en
el otro todas las mujeres.

7.21 Ejercicio. (∗) Se dice que una mano de dominó tiene falla si alguno de los
números entre el 0 y el 6 no aparece en la mano (cada número faltante es una falla); por
ejemplo la mano {2|1, 5|5, 3|1, 0|0, 1|0, 5|6, 0|2} tiene falla a 40 s. ¿Cuál es la probabilidad de
que una mano de dominó no tenga falla?

7.22 Ejercicio. (∗) Una persona quiere apostar que la suma de lo que muestren dos
dados es cierto número. ¿A qué número le conviene apostar?

7.23 Ejercicio. (∗) Se eligen al azar n cartas de la baraja. ¿Cómo debe ser n para que
la probabilidad de que entre las cartas elegidas haya (al menos) dos del mismo número sea
mayor que 21 ? ¿Cuál es la probabilidad si n = 14?

7.24 Ejercicio. (∗) Calcular la probabilidad de que al lanzar tres veces dos dados, las
tres veces los números que salgan sean iguales entre sı́.

7.25 Ejercicio. (∗) Se escogen al azar en sucesión tres números (posiblemente iguales)
entre el 1 y el 100. ¿Cuál es la probabilidad de que se hayan escogido en orden creciente
estricto?

7.26 Ejercicio. (∗) Un dado se lanza al aire 6 veces. ¿Cuál es la probabilidad de que
aparezca cada uno de los seis números una vez?

7.27 Ejercicio. (∗) Supongamos que de un grupo de 10 enfermedades cada una tiene
1
probabilidad 10 de atacar a un animal determinado a lo largo de su vida. ¿Qué probabilidad
tiene ese animal de enfermarse de al menos una de esas enfermedades?

7.28 Ejercicio. En cierto examen de opción múltiple con 5 opciones en cada respuesta
se califica como sigue: por cada respuesta correcta se otorga +1 punto, por cada respuesta
incorrecta se otorga − 14 de punto, y por cada pregunta sin contestar se otorgan 0 puntos.
¿Qué calificación esperarı́a obtener alguien que contestara todo el examen?

40
8. Principio de las Casillas

A primera vista el Principio de las Casillas (también llamado Principio de los Palo-
mares) parece extremadamente simple e inofensivo. Sin embargo tiene muchas aplicaciones
en problemas de del tipo que nos interesan en estas notas.

8.1. Principio de las Casillas. Si se dispone de n casillas para colocar m objetos y


m > n, entonces en alguna casilla deberán colocarse por lo menos dos objetos.

Empezaremos ilustrando con un ejemplo muy simple y tı́pico de aplicación varias veces
del Principio de las Casillas.

8.2 Ejemplo. Un costal está lleno de canicas de 20 colores distintos. Al azar se van
sacando canicas del costal. ¿Cuál es el mı́nimo número de canicas que deben sacarse para
poder garantizar que en la colección tomada habrá al menos 100 canicas del mismo color?
Solución. Notemos que si sacáramos 20 canicas, podrı́a ser que todas fueran de colores
distintos, ası́ que sólo podrı́amos garantizar que hay dos canicas del mismo color si sacáramos
21 canicas (aquı́ se aplicó el Principio de las Casillas). De la misma manera, necesitarı́amos
41(= 20 × 2 + 1) canicas para poder afirmar que con seguridad hay 3 canicas (al menos)
del mismo color, pues con 40 canicas podrı́a ser que cada color apareciera exactamente 2
veces. Con el mismo razonamiento que hemos seguido llegamos al resultado: Se necesitan
20 × 99 + 1 = 1981 canicas. ♦

8.3 Ejercicio. En un papel cuadriculado de 6×9 cuadrados se consideran 25 triángulos


arbitrarios y diferentes que tienen sus vértices en los puntos de intersección de las lı́neas de
la cuadrı́cula. Mostrar que no importa como se elijan los triángulos, forzosamente habrá (al
menos) dos triángulos con (al menos) un vértice en común.

Los siguientes ejemplos son todos de naturaleza muy distinta; sin embargo, en la solución
de todos ellos utilizamos el Principio de las Casillas.

8.4 Ejemplo. Algunos de los cuadritos de una cuadrı́cula de 3 × 7 se pintan de negro


y los otros se dejan en blanco. Probar que forzosamente las lı́neas de la cuadrı́cula forman
un rectángulo en cuyas cuatro esquinas los cuadraditos tienen el mismo color (los cuatro
blancos o los cuatro negros).
Solución. Supongamos que tenemos una cuadrı́cula pintada de manera tal que no se
forma el rectángulo con las esquinas del mismo color. Simbolicemos por N al color negro
y por B al color blanco, y observemos que los cuadritos de una columna pueden haber
quedado pintados según las siguientes 8 posibilidades: p1 = N N N , p2 = N N B, p3 = N BN ,
p4 = BN N , p5 = N BB, p6 = BN B, p7 = BBN y p8 = BBB. Supongamos que una de las
columnas está pintada según la posibilidad p1 ; entonces con cualquiera de las posibilidades
en que la columna tiene dos N 0 s se formará un rectángulo con las esquinas negras, ası́ que

41
ninguna columna está pintada ası́; pero entonces las columnas están sólo pintadas según las
posibilidades p1 , p5 , p6 , p7 y p8 ; como el número de columnas es 7, entonces el Principio de las
Casillas nos dice que debe haber dos columnas iguales, pero aquı́ también, por el Principio
de las Casillas, como son tres cuadritos en cada columna y sólo dos colores, hay un color
que se repite, y entonces es obvio que se forma un rectángulo con las esquinas del mismo
color. Concluimos entonces que la posibilidad p1 no aparece. Lo mismo ocurre al considerar
la posibilidad p8 . Entonces ninguna de las posibilidades p1 y p8 aparece; pero ası́ sobran
sólo 6 posibilidades, con lo cual, otra vez aplicando el Principio de las Casillas, tenemos dos
columnas iguales, y de ahı́ una contradicción. ♦

8.5 Ejercicio. Algunos de los cuadritos de una cuadrı́cula de 19 × 4 se pintan de rojo,


otros de azul y otros de verde (no se deja ninguno en blanco). Probar que forzosamente las
lı́neas de la cuadrı́cula forman un rectángulo cuyas cuatro esquinas tienen el mismo color.

8.6 Ejemplo. Probar que en cualquier conjunto de 6 personas forzosamente hay 3 que
se conocen todas entre sı́ o 3 tales que ninguna conoce a las otras 2.
Solución. Por cada una de las personas pongamos un punto e indiquemos que dos personas
se conocen poniendo una lı́nea entre los puntos que las representan, obteniendo ası́ la gráfica
de conocidos. Si A es una de las personas, las restantes 5 se pueden dividir en dos grupos:
las conocidas de A y las desconocidas de A. Por el Principio de las Casillas, alguno de los
grupos tendrá 3 o más elementos. Primero supongamos que el de los conocidos de A tiene
tres o más elementos y sean B1 , B2 y B3 conocidos de A.

A
........
... ..........
... .... ............
... ... .......
..... ... .......
.......
... ... .......
... ...

.
.
.
..
.
.
..
.
.
.
.
.
...
. B2
...... .... ....
.......
.......
.......
.......
.
... .... ... .... ....
.... .... ............
...
. ..
. .... .... .... .......
.
... .
.... ............
..
... .. ...
. .... .... .... ....
... .. .... .. .... .... .... ....
..
.... .
..
........... .... .... .... .... .... .. .. ...
. .... ....
B3
B1

Si dos de B1 , B2 y B3 se conocen entre sı́, entonces A junto con esos dos formará el grupo
de los tres conocidos que buscábamos. Si no, entonces B1 , B2 y B3 formarán el grupo de los
tres desconocidos que querı́amos. El caso en que el número de desconocidos de A sea 3 o más
se trata de manera análoga al anterior, considerando tres desconocidos C1 , C2 y C3 de A. ♦

El último ejemplo de esta sección nos servirá, además de para ver una aplicación intere-
sante (y muy simple) del Principio de las Casillas, para recordarnos algunas propiedades
importantes de los números. Recordemos que los números reales son aquéllos que nos sirven
para “medir con dirección”, es decir, aquéllos que se pueden representar en una recta una
vez que se ha establecido un punto de partida (el 0) y se ha indicado otro punto distinto (el
1) que nos señala precisamente la medida 1 y que la dirección del 0 al 1 es la positiva; ası́,

42
los números reales del mismo lado del 0 que el 1 son los positivos, mientras que los reales
del lado opuesto son los negativos. Entonces tenemos que todo
√ número entero es real, pero
hay reales que no son enteros, como por ejemplo −1.5, 2.47, 2 y π.
negativos
....................................................
positivos
.......................................................
..

| | | √| | |
........................................................................................................................................................................................................................................................................................................................................................................................................................................

−3 −1.5 0 1 2 2.47 π

El conjunto R de los números reales se descompone en dos conjuntos ajenos (es decir,
dos conjuntos sin elementos en común): el de los números racionales (denotado por Q) y el
de los números irracionales (denotado por I). Los números racionales son los que se pueden
expresar como cociente de enteros, y los irracionales son los que no. Por ejemplo, son números
racionales −1.5, 0, −3 y 2.47, pues se pueden expresar, respectivamente, como −3 , 0 , −3 y 100
247
.
√ 2 1 1
Utilizando resultados de divisibilidad de enteros no es difı́cil probar que 2 no es racional.
Un resultado conocido pero difı́cil de demostrar es que π tampoco es racional. La escritura
conocida con cifras de un número real cualquiera se llama expansión decimal del número
(o sea, la expansión decimal del número 23 053.04 es, precisamente, 23 053.04). La razón
para esta nomenclatura es que la expresión se basa en un sistema posicional en que en cada
posición la cifra (que puede ser cualquier entero entre 0 y 9) representa el número de veces
que debe tomarse la potencia de 10 correspondiente a esa posición; por ejemplo:
1
23 053.04 = 2 × 104 + 3 × 103 + 5 × 101 + 3 × 100 + 4 × 2 .
10
No debemos pasar por alto que algunas expansiones decimales son infinitas (después del
punto decimal), como por ejemplo la expansión de 13 , que consta de una infinidad de 30 s.
Esto usualmente se representa por 13 = .3, indicando que el 3 debe repetirse una infinidad de
veces. De esta manera, también podemos considerar números reales como 1.2403 en donde,
a partir del 2 la repetición es de las tres cifras 403. En los casos que acabamos de considerar
en que hay una repetición infinita de cifras, decimos que la expansión de los números es
periódica y llamamos periodo a lo que se repite (en el primer caso el periodo es 3 y en
el segundo, el periodo es 403). Podemos decir que las expansiones finitas son periódicas con
periodo 0 (por ejemplo, 2.4 = 2.40). Es importante notar que hay números reales que no
tienen expansión periódica, como por ejemplo el número .1010010001..., en el que después
del punto los 10 s están intercalados entre cada vez más 00 s. No es difı́cil probar que: Si un
número real tiene una expansión periódica, entonces el número es racional: Si el número se
llama x, lo podemos multiplicar por potencias apropiadas de 10, de tal manera que al restar
una de otra se elimine el periodo, y después despejar x de una expresión de enteros (por
ejemplo, si x = 3.825, entonces 103 x − 10x = 3825.25 − 38.25 = 3787, ası́ que x = 3787 990
). El
contenido del siguiente ejemplo, que demostraremos utilizando el Principio de las Casillas,
es, precisamente, el recı́proco de este resultado.

8.7 Ejemplo. Probar que la expansión decimal de cualquier número racional es pe-
riódica.

43
Solución. Sea ab el número considerado, donde a y b son enteros y b 6= 0. Al hacer la
división según el algoritmo usual, los residuos que van quedando son enteros entre 0 y b − 1,
ası́ que forzosamente deberá haber alguna repetición; a partir de ese momento, los cocientes
y los residuos que se van obteniendo van formando un periodo de repetición. ♦

8.8 Ejercicio. Sea A un conjunto de 19 enteros (distintos) escogidos dentro de las


sucesión aritmética 1, 4, 7, . . . , 100. Probar que en A hay dos elementos (distintos) cuya suma
es 104.

8.9 Ejercicio. La suma de 5 enteros positivos es 100. Probar que al menos hay dos de
esos números cuya diferencia es menor o igual que 9.

8.10 Ejemplo. Sea p 6= 2, 5 un número primo. Probar que hay una infinidad de múlti-
plos de p en la sucesión (1, 11, 111, 1111, . . .).
Solución. Sea ar el número formado por una cantidad r de 10 s. Gracias al Principio de las
Casillas, en la lista de residuos módulo p de la lista infinita 1, 11, 111, 1111, . . . debe haber
repetición, es decir existen r < s naturales tales que ar ≡ as (mod p), de donde as − ar es
múltiplo de p. Pero as − ar = 10r as−r y p es primo relativo con 10, ası́ que as−r es múltiplo
de p. Hasta aquı́ hemos encontrado un múltiplo de p dentro de la lista. Para encontrar una
infinidad observemos que el número formado por una cantidad múltiplo de s − r de 10 s
también es múltiplo de p (para ver esto basta aplicar el conocido algoritmo para hacer una
división que aprendimos desde niños). ♦

8.11 Ejemplo. Sean a1 , a2 , . . . , a10 enteros. Probar que existen

1 , 2 , . . . , 10 ∈ {−1, 0, 1}

no todos cero tales que 1 a1 + 2 a2 + · · · + 10 a10 es múltiplo de 1000.


Solución. Consideremos {1 a1 + 2 a2 + · · · + 10 a10 : i ∈ {0, 1}}. Las posibilidades para
expresar estos elementos son 210 = 1024; como hay sólo 1000 residuos posibles módulo 1000,
entonces, por el Principio de las Casillas, tenemos que dos de éstos son congruentes módulo
1000, de donde su diferencia es múltiplo de 1000. Ahora sólo observemos que la diferencia
es una expresión como la que estamos buscando. ♦

8.12 Ejercicio. (∗) En un torneo de futbol cada equipo juega una vez exactamente
con cada uno de los demás. Los juegos están se realizan siempre los domingos. Probar que
cualquier lunes siempre hay dos equipos que han completado exactamente el mismo número
de juegos.

8.13 Ejercicio. (∗) Probar que si cada punto del plano se colorea de rojo o azul,
forzosamente habrá un segmento de longitud 1 cuyos extremos tengan el mismo color.

44
8.14 Ejercicio. (∗) En un congreso internacional se reúnen n cientı́ficos de 6 paı́ses.
Durante el congreso los cientı́ficos se dividen en cuatro secciones de tal manera que dentro
de cualquier grupo de 6 participantes de la misma sección siempre hay dos cientı́ficos de la
misma edad. Encuentra el mı́nimo número n para el cual, bajo las condiciones mencionadas
arriba, se puede asegurar que existen tres cientı́ficos de una misma sección que tienen la
misma edad y pertenecen al mismo paı́s.

8.15 Ejercicio. (∗) Sea M un conjunto de 9 enteros ninguno de los cuales tiene un
divisor primo mayor que 6. Probar que hay dos elementos de M cuyo producto es un cua-
drado.

8.16 Ejercicio. (∗) Isabel escoge 8 puntos de los marcados. ¿Cuál es la probabilidad
de que cuatro de los puntos escogidos sean los vértices de un rectángulo?

• • • •
• • • •
• • • •

45
9. Coloración

En varios de los problemas de esta sección se estudia la posibilidad de cubrir tableros de


un determinado tamaño o forma con fichas que cumplen ciertas caracterı́sticas. Se entiende
que las fichas no deben traslaparse ni cubrir área fuera del tablero. Se pide además que
entre todas las fichas se tape todo el tablero. Para ilustrar, consideremos el caso de cubrir un
tablero de 8×8 con fichas de tamaño 2×1. Es muy fácil lograrlo y en la figura esquematizamos
2 formas de hacerlo.
.................. .................. .................. .................. .................. .................. ..................
.... ....
... ...
... ...
.................. .................. .................. .................. ... .................. ... .................. . .
... ...
.. ..
.................. .................. .................. .................. ... ... ..................
..... ..... ..... .....
... ... ... ...
... ... .. ...
.................. .................. .................. .................. .. . . .. .. .. . .
... ... ... ...
.. .. .. ..
.................. .................. .................. .................. ... .................. ... ... ... ..................

.................. .................. .................. .................. .................. .................. .................. ..................

.................. .................. .................. .................. .................. .... .................. .................. ....


... ...
.. ..
.................. .................. .................. .................. .................. .. .................. .................. ..

Desde luego, el ejemplo anterior fue muy sencillo y no tuvo ningún interés teórico. Sin
embargo una ligera variación lo vuelve mucho más interesante, como veremos a continuación.

9.1 Ejemplo. Utilizando fichas de tamaño 2 × 1, ¿es posible cubrir un tablero de 8 × 8


del que se han quitado dos esquinas opuestas (de 1 × 1)?
Solución. Podrı́amos hacer muchos intentos sin lograr cubrir el tablero; sin embargo esto
no nos demostrarı́a que no es posible. Utilizamos un argumento ingenioso: pintar el tablero
como si fuera un tablero de ajedrez (alternando los colores blanco y negro). Observamos
entonces que a las dos esquinas que se eliminaron les correspondı́a el mismo color, ası́ que
quedaron más cuadros de un color que de otro. Por otro lado, sin importar el lugar donde se
coloquen las fichas de 2 × 1, cada una de ellas cubre un cuadro negro y uno blanco ası́ que,
si se pudiera cubrir el tablero, el número de cuadros blancos cubierto serı́a el mismo que el
de negros. Este argumento demuestra que no es posible cubrir el tablero. ♦
...............................................................................................................................................................
... ....................................... ...................................... ...................................... ...
... ....................... ....................... ........................ ...
................................................................................... ........................................ ..........
.......................... ............................ .................... ....................
.......................... ............................ .......................... ..........................
................... ..................... .......................... ..........................
........................................................................................................................
................................... .......................... ............................ ....................
............................ . .................. . .................... ..........................
............................ ............................................................. ...... .......................... ......
................................. ............................ .................... ....................
.......................... ............................ .......................... ..........................
.......................... ............................ .......................... ..........................
. . . . . . . . . . . . . . ............ ......
................................... ......................... ........................... ..........................
.
.
......................... .
........................... . . . .
................... . .
............................ . . . . . . . . .
............................ .........................
. ...........................
.
...................
. . . . . . .
....................................................................................................
...... ....... ................................. .................................
................................. ...................................
.......................... ..................... .......................... ..........................
................................................................................................................
................................... . .................. . .................... ....................
............................ ...................
. ......
....................
. ....... . .................
.
..................... .................... ..................... ..........................
......................................................................................................................................
... ....................................... ....................................... .................................... ...
... ................................ ................................ .............................. ..
...........................................................................................................................................................

46
9.2 Ejemplo. A un tablero cuadriculado de 9×9 se le han quitado tres de sus esquinas
(de 1 × 1). ¿Es posible cubrirlo con fichas de 3 × 1?
Solución. Utilizamos ahora 3 colores que indicamos en la figura con los números 1, 2 y 3.
........................................................................................................................................
.... .... .... .... .... .... .... ...
....
.. 2 ....
.. 3 ....
.. 1 ....
.. 2 ....
.. 3 ....
.. 1
....
..
.... 2
....................
...
3 1 2 3 1 2 3 1
.....................
...
.. 2
...
2 3 1 2 3 1 2 3
......................
...
. 1
...
1 2 3 1 2 3 1 2
.....................
3
...
.
...
3 1 2 3 1 2 3 1
.....................
2 ...
.
...
2 3 1 2 3 1 2 3
.....................
1 ...
..
...
1 2 3 1 2 3 1 2
.....................
3 ...
.
...

.
3 .
1 .
2 .
3 . .
1 .
2 .
3 1
....................
2 ...
..
.... .... .... .... .... .... .... .... ...
...
.2 3
...
.
...
. 1 ...
. 2 ...
. 3...
.
...
.1 ...
. 2
..................................................................................................................................................................
....
. 3

En este caso, cualquier colocación de las fichas de 1×3 cubre una vez cada uno de los colores,
ası́ que en la coloración deberı́a haber la misma cantidad de cuadros de cada color; pero esto
no es cierto ya que en la cuadrı́cula de 9 × 9 sı́ habı́a la misma cantidad de cuadros de cada
color, pero a dos de los tres cuadros que se quitaron les correspondı́a el mismo color (el 1).
(notemos que hay que escoger la coloración en forma adecuada; por ejemplo, si se hubiera
quitado la esquina inferior izquierda en lugar de la derecha, la coloración dada no servirı́a
pues se quitarı́a un cuadro de cada color, en ese caso hubiera convenido comenzar el segundo
renglón con el color 2 y después seguir alternando de la misma manera.) ♦

9.3 Ejercicio. Rehacer el ejemplo anterior utilizando sólo dos colores.

9.4 Ejercicio. (∗) ¿Es posible construir un rectángulo con las cinco fichas que se
muestran a continuación (todos los cuadritos marcados son de 1 × 1)?

.................... ......................
...
.... .....
.. . .....
.................... ................... .................... ... .
.. ...
..
. ....
.................... ...................

9.5 Ejercicio. (∗) La estrella que se muestra abajo consta de 108 triángulos equiláteros
iguales. ¿Es posible cubrirla con fichas de 4 triángulos equiláteros (del mismo tamaño que
los de la estrella) como la que se muestra al lado de la estrella?

47
..
......
... ...
... ...
..............................
.. .......
... ...
... .... ..... .....
... . . .
..........................................................
.
... ... ...... .. ..
. ... .... ..... ..... ..... .....
... . ...
... . ... ...
.............................................................................................................................................................................................................................................
... .. . . . .. . . .
... .... ..... ..... ..... ..... ..... ..... ..... .... ..... ..... ..... ..... ..... ..... ..... .....
... ... ... .. ... .. ... ... ... .. ... .. . . . . . .
.......................................................................................................................................................................................................................
... .... ....
. . ... ..... .... .... .... .
... ... . .
. . .. . ... . . . ..
. . .
.. . ...
... .... ..... ..... ..... ..... ..... .... ..... ..... ..... ..... ..... ..... ..... .....
... ... . . . . . . . . . . . . . .
..........................................................................................................................................................................................................
... .. . .. .. .. ..
... ..... ..... ..... ..... .... ..... ..... ..... ..... ..... ..... ..... .....
... .. ... ... ... ... ... .. ... .. ... .. ... .. ............................................................
... ... . . . . . . . . . . . .
........................................................................................................................................................................... ... ... . . .
.
. . . . . . .. ..... ... ..... ...
... ...
.
... ... ... ...
.
... ...
.
.. ...
. .
.. ... ... ... .. .. ..
... ..... .... ..... .... .... .... ..... ..... .... ..... ..... .... ..... ....................................................
..
... ...... ...... ... .. ..... ... .. ...... ..
.................................................................................................................................................................................
. .. . . . . .. . . . .
...... .. . . . . . . . .. . .
... ... ... ... .. ... ... ... ... ... ... ... ... ... .. ...
.... .... ..... ..... ..... ..... ..... .... .... ..... .... ..... ..... ..... ..... .....
.
..
. .........................................................................................................................................................................................................
.. ... .. .. .... ... .. .. ....
... .... ... ... ... .... ... .... ... ... ... ... ... .... ... .... ... ...
... ..... ..... ..... ..... ..... ..... ..... .... ..... ..... ..... ..... ..... ..... ..... .... .....
... .. .. .. .. .. .. .. .. .
...........................................................................................................................................................................................................................................
... .. .. .
... .... ..... .... ..... .....
... ... ... .... . .
.
..........................................................
...
... ....... ...
... ..... ..... .....
... .. . .
................................
... .
... .....
... ...
......
..

9.6 Ejercicio. (∗) Se considera el conjunto S de puntos del espacio formado por los
vértices de un cubo y los puntos medios de las caras del cubo. Se ponen segmentos entre los
puntos de S uniendo cada punto medio de una cara con los 4 vértices del cubo que forman
esa cara. ¿Es posible hacer un recorrido sobre los segmentos dibujados que pase exactamente
una vez por cada uno de los puntos de S?

9.7 Ejemplo. Según un diseño ya establecido, un piso rectangular se va a cubrir con


algunos mosaicos de 2×2 y otros de 1×4. Se rompió un mosaico y el distribuidor lo sustituyó
por otro del tipo contrario. Probar que no se puede rediseñar el arreglo.
Solución. Cuadricular el piso, y en los renglones pares colorear alternadamente con negro
y blanco. Dejar los renglones impares en blanco (ver figura).
.... .... .... .

.... .... .... .


...........................................................................
. .
............................................................................ ................................................................... ....................................................................
............................................................. .............................................................. ................................................................ ................................................................
.......................................................... .............................................................. ................................................................ ................................................................
.................................................................. ............................................................... .................................................................. ...................................................................... .... .... .

......................................... ............................................. .................................................. .......................................................... .... .... .


................................................................ ................................................................... .................................................................. ................................................................
.................................................................. ................................................................ .................................................................. .................................................................
.................................................................. .................................................................. ................................................................ ...............................................................
............................................ ............................................. .................................... ............................................ .... .... .

... ... ... ... ... ... ... ...... .... .... .
... ... ... ... ... ... ... ...
... ... ... ... ... ... ... ...
. . . . . . . .

Entonces, según el primer diseño, cada mosaico de 2 × 2 cubrirı́a exactamente un cuadro


negro, mientras que los de 1×4 cubrirı́an 0 o 2 y ası́ la paridad de los cuadros negros cubiertos
serı́a distinta al cambiar un mosaico. ♦

Para ver que sı́ es posible lograr exactamente 9 cuadros vacı́os, en el siguiente dibujo
hemos indicado con una lı́nea diagonal el que las fichas que están en las casillas que une la
diagonal se intercambian, dejando entonces vacı́as las casillas indicadas con •.

48
9.8 Ejercicio. (∗) Probar que si hay n puntos en el plano, con n ≥ 4, entonces es
posible colorearlos con dos colores de manera que no exista ninguna recta que deje todos los
puntos de un color de un lado y los del otro color del otro lado (y que no toque a ninguno
de los puntos).

Los siguientes ejemplos tratan coloraciones de cantidades infinitas de puntos.

9.9 Ejemplo. Probar que si cada punto del plano se colorea de rojo o azul, forzosa-
mente habrá un segmento de longitud 1 cuyos extremos tengan el mismo color.
Solución. Primera forma. Tomemos un triángulo equilátero de lado 1. Dos de sus vértices
deben ser del mismo color.
Segunda forma. Tomemos un punto, digamos, rojo y consideremos el cı́rculo con centro
en ese punto y radio 1. Si en el cı́rculo hay un punto rojo, ya acabamos; si no, entonces todo
el cı́rculo es azul y en él es claro que hay una cuerda de longitud 1. ♦

9.10 Ejemplo. Probar que si cada punto del plano se colorea de azul, rojo o verde,
entonces uno de los tres colores tiene una pareja de puntos a distancia 1.
Solución. Supongamos que no. Entonces todo triángulo equilátero de lado 1 tiene sus
tres vértices de distinto color. Tomemos uno de estos triángulos y sea O un vértice rojo.
Del otro lado de la base que tiene por extremos un punto verde y otro azul pongamos otro
triángulo; entonces el nuevo vértice debe ser también rojo. Esto es cierto para cualquier
triángulo equilátero uno de cuyos vértices sea O, es decir, el cı́rculo con centro O
√ y radio 1
tiene tiene todos sus puntos azules o verdes y el cı́rculo con centro O y radio 3 es todo
rojo. Es claro que este cı́rculo tiene una cuerda de longitud 1 y ası́ encontramos dos puntos
rojos a distancia 1. ♦

9.11 Ejercicio. (∗) Cada punto del espacio es rojo, azul, blanco o verde. Probar que
hay un segmento unitario cuyos extremos son del mismo color.

49
10. Gráficas

Una gráfica (finita, simple) G (consta de un conjunto (finito) V = V(G) cuyos elemen-
tos se llaman vértices, y de otro conjunto A = A(G) que consta de subconjuntos de dos
elementos de V llamados aristas. Escribimos G = (V, A). Si a = {u, v} ∈ A, escribimos
a = uv y decimos que u y v son extremos de a.

Por el momento sólo trabajaremos con este tipo de gráficas y dejaremos para después el
estudio de multigráficas en donde se permite que A sea multiconjunto (es decir, que entre
dos vértices haya más de una arista), o de seudográficas en las que, además, se permite que
las aristas consten de un solo elemento de V (una arista ası́ se llama lazo), o el de digráficas
(o multi/seudo digráficas) en las que las aristas son parejas ordenadas de vértices. En este
curso no trabajaremos con gráficas infinitas (aquéllas en las que el conjunto de vértices es
infinito).

En general se hace una representación geométrica de la gráfica poniendo un punto por


cada vértice y representando cada arista por una lı́nea entre los dos vértices que la determinan
(en el caso de digráficas se pone una flecha).

De aquı́ en adelante G denota una gráfica cualquiera con conjunto de vértices V y conjunto
de aristas A. Los vértices de G serán u, v, w, . . ..

Si v es un vértice, el grado de v es el número δ(v) de aristas que tienen como elemento


a v. Definimos ∆(G) = max{δ(v) : v ∈ V} y δ(G) = min{δ(v) : v ∈ V}. La gráfica G es
r-regular si ∆(G) = r = δ(G).

10.1 Proposición. La suma de los grados de los vértices de una gráfica es el doble del
número de aristas.
Demostración. Al sumar los grados de los vértices, cada arista se cuenta dos veces (una
por cada uno de sus extremos). ♦

10.2 Ejercicio. Probar que el número de personas en el mundo que tienen un número
impar de hermanos es par.

10.3 Ejercicio. (∗) Sea G una gráfica regular. Si G tiene 26 aristas, ¿cuántos vértices
puede tener?

10.4 Ejercicio. (∗) Se quiere diseñar una competencia con n participantes en la que
cada uno compita exactamente con otros k. Probar que una competencia tal puede diseñarse
si y sólo si nk es par y n ≥ k + 1.

Si a = uv ∈ A, decimos que u y v son adyacentes y que a es incidente a u y a v. Si

50
dos aristas tienen exactamente un vértice en común decimos que son adyacentes.

Dados u, v ∈ V, un camino C de u a v (o uv-camino) es una sucesión de vértices


alternados con aristas C = (u = v0 , a1 , v1 , a2 , . . . , an , vn = v) tal que para cada i = 1, . . . , n, ai
es incidente a (los vértices distintos) vi−1 y vi . También escribimos C = (u = v0 , v1 , . . . , vn =
v) o C = (a1 , . . . , an ). El camino es cerrado si v0 = vn ; si no, se dice que es abierto. El
camino es trayectoria si no repite vértices; es paseo si no repite aristas y es ciclo si no se
repiten aristas y los únicos vértices que se repiten son el primero y el último (v0 = vn ). La
longitud del camino, l(C), es n (el número de aristas).

10.5 Ejercicio. (∗) Probar que todo uv-camino C contiene una uv-trayectoria. (Suge-
rencia: Proceder por inducción sobre la longitud de la trayectoria.)

Una gráfica es conexa si dados cualesquiera dos vértices u y v existe un uv-camino (o


uv-trayectoria). En caso contrario se dice que la gráfica es disconexa.

Dos gráficas G1 = (V1 , A1 ) y G2 = (V2 , A2 ) son isomorfas si existe una biyección f :


V1 → V2 tal que uv ∈ A1 si, y sólo si, f (u)f (v) ∈ A2 . Cuando dos gráficas son isomorfas,
las consideramos iguales; ası́ por ejemplo podemos hablar de la gráfica que consta de un solo
vértice. Es claro que dos gráficas isomorfas comparten las mismas propiedades, por ejemplo
de conexidad o del grado de los vértices (más aún, un isomorfismo debe mandar un vértice
a otro del mismo grado). Denotamos por Tn la trayectoria de longitud n, y por Cn al ciclo
de longitud n.

Decimos que una gráfica H es subgráfica de G (o que está contenida en G) si el


conjunto de vértices y de aristas de H son subconjuntos del conjunto de vértices y del de
aristas de G, respectivamente (o si hay un isomorfismo entre H y una subgráfica de G).
Una componente conexa de G es una subgráfica conexa maximal (es decir, ella es conexa,
pero cualquier otra subgráfica que la contenga propiamente es disconexa). La componente
conexa de un vértice v es la subgráfica conexa maximal que lo contiene.

10.6 Ejemplo. Hay 6 gráficas conexas no isomorfas con 4 vértices.


Demostración. Son las siguientes 6:

•...... • •
...............................
.....
... ...
•..............................•........ •.........................................•........... •......................................................•..........
... .....
•........................•.........................•..........................•.. ..
...
. ...
..
. ...
...
...
...
...
..
....
.
...
... .........
.. ......
.. ...
...
..
... ....... ..
... .............. ...
.. ...... ...... ...
• • •
...................................................... • •
.............................. • •
.............................
• ................................ • • .................................... • ♦

51
10.7 Ejercicio. (∗) Dar un ejemplo de dos gráficas conexas no isomorfas en las que
los grados de los vértices sean los mismos.

La gráfica completa con n vértices es aquélla en la que por cada par de vértices hay
una arista que los une (es decir, tiene n2 aristas); denotamos esta gráfica por Kn . Si G es
una gráfica con n vértices, la podemos considerar como subgráfica de Kn y el complemento
G de G es la subgráfica de Kn formada por los mismos vértices de G pero en el que las aristas
son aquellas aristas de Kn que no son aristas de G.

•....... • .....
•......................................................................................•.......... • ...........
........ ........
...... .. ... .....
... .... ..... .... .... ...........
... ..... . . ...
.... .. ... .... ......... ..... ....
. ...... .. ..... ......
.
..... ..... .. ......
...
..
...
...
...
...
...
...
..
.....
..... ........
..... .....
...
... • ... ....... . ... .... .•
.................................................................................................
• ...
...
...
.
...
.
.
... ...
...
...
...
...
... .....
. ..
..
.
..
............
.....
.
...
...
...
... ........ ....
...
...
.......
... ........... ...
............. ...............
.
.......
..
...
...
.
..
...
... .........
.. .
.....
..... .... ... .. ... ... . ..
..
. ... ... ... .................... .... ....
... .. ...
.. ... ....... ..... ...
..... .. .
... . ..... .
. .
...... .. .. .
... . .......... ............. ... . .
• . • •
....................................................................
.
• ............................................................ .
• • •
..................................................

K1 K2 K3 K4 K5

La subgráfica inducida por un subconjunto V 0 del conjunto de vértices es (V 0 , A0 ),


donde si u, v ∈ V 0 y uv ∈ A entonces uv ∈ A0 . Una subgráfica generadora es aquélla que
tiene todos los vértices de G.

En demostraciones por inducción, muchas veces conviene considerar las gráficas obtenidas
de la original al quitar un vértice o una arista. Si v ∈ V, la subgráfica de G que tiene por
conjunto de vértices a V \ {v} y por conjunto de aristas a A \ {a ∈ A : v ∈ a} se denota
por G − v. Si a ∈ A, la subgráfica (V, A \ {a}) de G se denota por G − a. También, si u y v
son vértices de G no adyacentes, la gráfica G + uv tiene por conjunto de vértices a V y por
conjunto de aristas a A∪{uv}. Análogamente podemos considerar G −S para S subconjunto
del conjunto de aristas o vértices.

10.8 Proposición. Si G es una gráfica entonces ella o su complemento es conexa.


Demostración. Primera forma. Supongamos que G no es conexa. Probaremos que cuales-
quiera dos vértices están conectados en G. Tomemos u, v ∈ V distintos. Si u y v están en
diferente componente conexa de G, entonces hay una arista entre ellos en G. Si u y v están en
la misma componente conexa de G, sea w un vértice en otra componente (que existe porque
estamos suponiendo que G no es conexa. Entonces, en G hay una arista de u a w y también
de v a w y, por tanto, (u, w, v) es una uv-trayectoria en G.

52
Segunda forma. Procedemos por inducción sobre el orden n de G. Para n = 1 el resultado
es obvio. Supongamos entonces que n ≥ 2. En Kn pintemos las aristas de G de verde y las
aristas de G de rojo. Si un vértice u tiene todas sus aristas de un mismo color, entonces u
está unido con todos los demás vértices en la gráfica de ese color, ası́ que el resultado es
cierto. Supongamos entonces que u tiene aristas de cada uno de los dos colores. Por hipótesis
de inducción, en Kn − u, alguna de las dos subgráficas resultantes G − u o G − u es conexa.
Pero a u le llegan aristas de ambos colores ası́ que está unido con las dos gráficas y, en
consecuencia, G o G es conexa. ♦

10.9 Ejercicio. Encontrar una gráfica G en la que ambas G y G sean conexas.

10.10 Ejercicio. Probar que sólo hay dos gráficas 4-regulares de orden 7. ¿Cuántas
gráficas 6-regulares hay de orden 9?

Una gráfica conexa sin ciclos se llama árbol. Un bosque es una gráfica cuyas compo-
nentes conexas son árboles. Un vértice de grado 1 es una hoja.

10.11 Lema. Un árbol tiene al menos dos hojas.


Demostración. Una trayectoria maximal (en el sentido de que no está contenida propia-
mente en ninguna otra trayectoria) tiene sus extremos de grado 1. ♦

10.12 Proposición. Una gráfica conexa con n vértices es árbol si, y sólo si, tiene n − 1
aristas.
Demostración. (⇒) Inducción sobre n. Si n = 1, el número de aristas es 0 = n − 1.
Supongamos n ≥ 1. Sea u vértice de grado 1. Entonces G − u es árbol con n − 1 vértices,
por tanto, por hipótesis de inducción, tiene n − 2 aristas, ası́ que el árbol que tenı́amos tiene
n − 1 aristas.

(⇐) Supongamos que la gráfica tiene k ≥ 1 ciclos y quitemos una arista de un ciclo (la gráfica
permanece conexa); repitamos esto hasta eliminar todos los ciclos; obtenemos un árbol con
n vértices y n − 1 − k < n − 1 aristas, lo cual es una contradicción. ♦

10.13 Ejercicio. Probar que las siguientes tres propiedades son equivalentes para una
gráfica conexa G:
(a) G es árbol.
(b) Si a G se le quita una arista cualquiera, la gráfica resultante es disconexa.
(c) Entre cualesquiera dos vértices hay exactamente una trayectoria.

10.14 Ejercicio. (∗) En cada uno de los casos siguientes explicar por qué no se puede

53
construir una gráfica G con las condiciones indicadas.
(a) Que los grados de los vértices sean (9, 6, 6, 4, 3, 3, 1).
(b) Que los grados de los vértices sean (7, 4, 4, 4, 3, 3, 2, 1, 1).
(c) Que sea árbol y los grados de los vértices estén dados por la sucesión (6, 5, 4, 2, 2, 1, 1, 1).

10.15 Ejercicio. (∗) ¿Cuántos árboles no isomorfos hay con 6 vértices.

10.16 Ejercicio. (∗) ¿Es posible encontrar dos árboles no isomorfos en que la sucesión
de grados sea la misma?

10.17 Ejercicio. Probar que toda gráfica conexa tiene árbol generador.

Decimos que G es bipartita si V se puede partir en dos conjuntos (ajenos) U y W, no


vacı́os, de manera que todo elemento de A tiene un extremo en U y otro en W. Para m y n na-
turales la gráfica completa bipartita, Km,n , consta de m+n vértices u1 , . . . , um , w1 , . . . , wn
y las mn aristas ui wj con i ∈ [m] y j ∈ [n].

10.18 Proposición. Una gráfica es bipartita si, y sólo si, no tiene ciclos de longitud
impar.
Demostración. (⇒) Coloreemos los vértices de uno de los conjuntos de rojo y el del otro
de azul. En cualquier ciclo los vértices están coloreados alternadamente, ası́ que no puede
haber ciclos de longitud impar.

(⇐) Sin pérdida de generalidad G es conexa. Supongamos que no tiene ningún ciclo de
longitud impar. Tomemos un vértice cualquiera v y pintémoslo de azul. Ahora pintemos
todos los vértices unidos a v de rojo; posteriormente pintemos todos los vértices unidos a
éstos de azul, etc. El que no haya ciclos impares nos dice que si un vértice ya está pintado
de un color no trataremos de pintarlo del color contrario. La coloración nos da la partición
buscada. ♦

Sea G conexa. Si u, v ∈ V, entonces su distancia es:


d(u, v) = min{l(T ) : T es uv−trayectoria}.
(Para gráficas no conexas podrı́a definirse d(u, v) = |V| cuando u y v pertenecen a diferente
componente conexa.)

54
10.1. Paseos eulerianos y ciclos hamiltonianos

Un paseo euleriano en una multigráfica es un paseo que pasa por todas las aristas de
G. Una multigráfica es euleriana si tiene paseo euleriano cerrado.

10.19 Teorema. Las siguientes son propiedades equivalentes para una multigráfica
conexa G.
(a) G es euleriana.
(b) Todo vértice de G tiene grado par.
Demostración. (a) ⇒ (b) Es claro, porque cada vez que se pasa por un vértice se usan
dos aristas.
(b) ⇒ (a) Probemos esto por inducción sobre el número de aristas. La demostración de
la base de inducción (0 aristas) es clara. Sea G una gráfica con a ≥ 1 aristas y 0 vértices
de orden impar. Tenemos la hipótesis de inducción siguiente: En una gráfica conexa con 0
vértices de orden impar y menos de a aristas se puede construir un paseo euleriano iniciando
en cualquier vértice y y cualquier arista incidente a él. Utilizando esta hipótesis construyamos
un paseo en G iniciando en cualquier vértice v. Para esto tomemos cualquier arista que tenga
un extremo en v y después una arista que inicie donde esa arista terminó; continuemos ası́
sucesivamente mientras podamos ir escogiendo aristas sin repetir. En el momento en que
nuestro paseo no pueda continuar será porque llegamos a un vértice en el que todas las
aristas ya se han usado; como todos los vértices tienen orden par, eso querrá decir que el
recorrido se cerró, es decir, que llegamos al vértice v (donde habı́amos empezado). En este
momento es posible que todavı́a no se hayan usado todas las aristas; sin embargo, cada una
de las partes conexas de la gráfica que no hayan sido recorridas es una gráfica en la que
todos los vértices son de orden par y que tiene menos aristas que nuestra gráfica. Entonces,
consideremos un vértice w del recorrido ya construido en el que haya alguna arista todavı́a
no usada y sea C la parte conexa de la gráfica que contiene a w y que todavı́a no ha sido
recorrida. Por la hipótesis de inducción, en C hay un recorrido euleriano que inicia en w.
Entonces a nuestro recorrido le intercalamos esa parte (que obviamente debe terminar en
el mismo w). Podemos hacer lo mismo en todos los vértices donde haya aristas no usadas
todavı́a, obteniendo ası́ el recorrido euleriano. ♦

55
10.20 Corolario. Una multigráfica G tiene paseo euleriano abierto si, y sólo si, tiene
exactamente dos vértices de grado impar.
Demostración. Agregamos arista entre esos vértices y, usando 10.19, construimos un paseo
empezando en una de esas aristas; después retiramos del paseo la arista agregada. ♦

10.21 Observación. De la demostración del teorema es claro que el paseo puede


empezar en cualquier arista si todos los vértices tienen grado par y en, el otro caso, en
cualquier arista de cualquier vértice de grado impar.

10.22 Ejercicio. (∗) El problema de los puentes de Köninsberg describe 4 trozos


de tierra A, B, C y D, separados por un rı́o pero unidos por algunos puentes, como se
muestra en el esquema. ¿Es posible pasar de pedazos de tierra a otros usando cada puente
exactamente una vez?

10.23 Ejercicio. (∗) (a) Sea G una gráfica regular en la que los vértices tienen grado
4. Probar que es posible colorear las aristas de G de manera que a cada vértice lleguen dos
aristas azules y dos rojas.
(b) Probar que si en G todos los vértices tienen grado 6, entonces no necesariamente es
cierto que se puedan pintar de manera que a cada vértice lleguen tres aristas azules y tres
rojas.

Ası́ como hablamos de caminos eulerianos, los cuales usan todas las aristas, podemos
considerar caminos hamiltonianos; son aquéllos que usan todos los vértices. Ası́ habla-

56
mos de ciclo hamiltoniano o de trayectoria hamiltoniana. Una gráfica G es gráfica
hamiltoniana si tiene ciclo hamiltoniano.

10.24 Ejemplo. Para n ≥ 3, Kn y Cn son hamiltonianas.

No existe respuesta completa sobre la existencia de ciclos hamiltonianos en una gráfica.


El siguiente es un criterio que podemos usar para determinar que ciertas gráficas no son
hamiltonianas.

Dada una gráfica G denotamos por k(G) al número de componentes conexas de G.

10.25 Proposición. Si G es hamiltoniana, entonces para todo S ⊂ V no vacı́o se tiene


que k(G − S) ≤ |S|
Demostración. Si C es un ciclo hamiltoniano en G entonces k(G − S) ≤ k(C − S) ≤ |S|. ♦

El recı́proco del criterio 10.25 es falso; en general se usa en forma negativa, es decir,
cuando en una gráfica es posible encontrar un subconjunto S del conjunto de vértices tal
que al quitarlo el número de componentes es más grande eque el número de elementos de S,
entonces se puede asegurar que la gráfica no es hamiltoniana. Ilustramos esto en es siguiente
ejemplo.
10.26 Ejemplo. La gráfica G cuyos vértices son los cuadros de un tablero de ajedrez
de 4 × 4 con una arista entre dos vértices si, y sólo si, los cuadros correspondientes se pueden
alcanzar con un salto de caballo no es hamiltoniana.
Solución. En el dibujo se ilustra un conjunto S con cuatro elementos tal que al quitar S
de la gráfica, el número de componentes es 6.

10.27 Teorema. (Teorema de Ore.) Si G es una gráfica con n ≥ 3 vértices en la que


para cada par de vértices u y w no adyacentes se tiene que δ(u) + δ(w) ≥ n, entonces G es
hamiltoniana.
Demostración. Supongamos que G no tiene ciclo hamiltoniano y agreguemos todas las
aristas que podamos sin que se forme un ciclo hamiltoniano. En la nueva gráfica la condición
de que la suma de grados de vértices no adyacentes es mayor o igual que n se satisface ası́
que podemos suponer que la misma G es tal que al agregarle cualquier arista se forma un
ciclo hamiltoniano. Sean u y w dos vértices entre los cuales no hay arista (existen porque en
Kn hay ciclo hamiltoniano). Como al agregar la arista uw se formarı́a un ciclo hamiltoniano,
eso quiere decir que hay una trayectoria hamiltoniana (u = v1 , . . . , vn = w). Ahora, como

57
δ(u) + δ(w) ≥ n, existe un ı́ndice i tal que uvi+1 y vi w son ambas aristas puesto que, en
caso contrario, consideremos la función f : N (w) → V \ N (u) dada por f (vi ) = vi+1 que es
inyectiva, y tenemos que N (u) ∩ Im(f ) = ∅ y que u ∈/ N (u) ∪ Im(f ), de donde
δ(u) + δ(w) = |N (u)| + |N (w)| = |N (u)| + |Im(f )| ≤ n − 1,
lo cual es una contradicción. Tomemos entonces un ı́ndice i tal que uvi+1 ∈ A y vi w ∈ A e
invertamos la porción de la trayectoria de vi+1 a w para obtener el ciclo:
(u = v1 , v2 , . . . , vi , w = vn , vn−1 , . . . , vi+1 , v1 = u),
de donde tenemos una contradicción. ♦

10.28 Ejercicio. A una comida van a asistir n ≥ 3 embajadores. Cada uno de ellos
es amigo de al menos n2 embajadores. Probar que se pueden sentar en una mesa redonda de
manera que cada embajador esté al lado de dos de sus amigos.

10.29 Ejercicio. (∗) Determinar si las siguientes gráficas (gráficas de los sólidos
platónicos) son hamiltonianas o no.

10.30 Ejercicio. (∗) Sea Gm,n la gráfica cuyos vértices son los cuadros de un tablero
rectangular de ajedrez de m × n con una arista entre dos vértices si, y sólo si, los cuadros
correspondientes se pueden alcanzar con un salto de caballo. Usar 10.25 para probar que si
m es impar, entonces Gm,m no es hamiltoniana.

58
10.31 Ejercicio. (∗) Decir cuál es el error en la siguiente “demostración” del problema
de los embajadores 8.6: “Procedemos por inducción sobre n. Para n = 3 es cierto pues n2 = 32 ,
ası́ que cada embajador debe tener al menos dos amigos y la gráfica de amistades es un
triángulo. Consideremos ahora el caso de n embajadores con n ≥ 4; usando la hipótesis de
inducción sentemos a n − 1 de los embajadores en mesa redonda y sea E el embajador que
falta por sentarse. Como él tiene al menos n2 amigos hay dos embajadores sentados juntos
que son amigos de E y entonces él se puede sentar entre ellos.”

10.32 Ejercicio. (∗) Hay un tesoro en cada cubo de 1 × 1 × 1 de los 343 que forman un
cubo de 7 × 7 × 7. Un duende se encuentra en el cubo central; en cada momento puede pasar
de un cubo a cualquier otro que comparta un cuadrado con el cubo donde está. Resulta que
si regresa a un cubo por el que ya pasó, entonces un monstruo le quita todos los tesoros que
ha obtenido hasta el momento. Las salidas están en las 8 esquinas del cubo. ¿Es posible que
salga del cubo con todos los 343 tesoros?

10.2. Gráficas planas

Decimos que G es una (multi)gráfica plana si está dibujada en el plano (o, equivalen-
temente, en la superficie de una esfera) y sus aristas no se intersectan (salvo en los vértices
que las forman).

Una (multi/seudo) gráfica G es aplanable si es isomorfa a una (multi/seudo) gráfica


plana.

10.33 Ejemplo. K4 es aplanable.

En una gráfica plana cualquier región delimitada por aristas es una cara, incluso la región
exterior (no acotada).

En dos gráficas planas isomorfas es posible que las caras no tengan el mismo número de
lados, como lo muestra la siguiente figura.

59
10.34 Ejemplo. Son aplanables Cn , los árboles y las gráficas de los sólidos platónicos.

10.35 Ejercicio. (∗) Probar que K2,n es aplanable.

En lo que sigue, denotamos por V , A y C a los números de vértices, aristas y caras,


respectivamente, de una (multi/seudo) gráfica plana G.

10.36 Teorema. (Fórmula de Euler.) Si G es conexa, entonces

V − A + C = 2.
Demostración. Inducción sobre A. Por ser la gráfica conexa, el menor número de aristas
que puede tener es A = V − 1, cuando G es un árbol. En este caso sólo hay una cara, de
donde se satisface trivialmente la fórmula. Ahora tomemos una gráfica G con A aristas, donde
A ≥ V . Entonces G tiene ciclos; sea a una arista perteneciente a un ciclo y consideremos la
gráfica G 0 obtenida de quitar a a G. Observemos que al quitar la arista a, alguna cara se une
con otra (todo esto, incluso si a es lazo o arista múltiple), de manera que

V 0 = V,
A0 = A − 1,
C 0 = C − 1,

donde V 0 , A0 y C 0 son, respectivamente, los números de vértices, de aristas y de caras de


G 0 , y ası́, usando la hipótesis de inducción, es claro que la fórmula se satisface también para
G. ♦

10.37 Corolario. El número de caras de una (multi/seudo) gráfica aplanable conexa


no depende del dibujo en el plano. ♦

10.38 Ejercicio. Usar la fórmula de Euler para determinar el número de regiones en


que n rectas en posición general dividen al plano.

10.39 Proposición. Si G tiene al menos 3 vértices y es gráfica aplanable máxima (es


decir, al agregar una arista ya no es aplanable) entonces todas sus caras son triángulos.
Demostración. Dibujemos G de manera plana en R2 , y supongamos que alguna cara tiene
más de 3 lados; entonces se puede agregar una arista entre dos vértices no consecutivos de
esa cara y la gráfica sigue siendo plana, lo cual es una contradicción. ♦

60
10.40 Proposición. Si G es gráfica aplanable con V ≥ 3 vértices y A aristas entonces

A ≤ 3V − 6.
Demostración. Construyamos G 0 a partir de G agregando aristas de manera que G 0 sea
plana máxima (y conexa). Si V 0 , A0 y C 0 son los números de vértices, aristas y caras de G 0 ,
0
respectivamente; entonces 3C2 = A0 (pues cada cara tiene tres aristas y cada arista pertenece
a dos caras), de donde C 0 = 32 A0 ; sustituyendo en la fórmula de Euler tenemos

2 1
2 = V 0 − A0 + A0 = V 0 − A0 .
3 3
Pero V = V 0 y A ≤ A0 , de donde
1 1
V − A ≥ V 0 − A0 = 2,
3 3
y de aquı́ tenemos el resultado buscado. ♦

10.41 Corolario. K5 no es aplanable.


Demostración. Si fuera aplanable cumplirı́a que A ≤ 3V − 6, pero 3V − 6 = 3 · 5 − 6 =
9 < 10 = A. ♦

10.42 Observación. Si H es subgráfica de una gráfica aplanable, entonces H es apla-


nable.

10.43 Corolario. Kn no es aplanable para n ≥ 5. ♦

10.44 Ejercicio. (∗) Probar que K3,3 no es aplanable usando el teorema de Euler.

10.45 Corolario. Si G es aplanable entonces tiene al menos un vértice de grado menor


o igual que 5. P
Demostración. Si todos tuvieran grado mayor o igual que 6, entonces 2A = v δ(v) ≥ 6V,
de donde A ≥ 3V > 3V − 6. ♦

10.46 Nota. Como acabamos de ver, K5 y K3,3 no son aplanables ni lo es ninguna


gráfica que contenga alguna de ellas como subgráfica. Una especie de recı́proco de este
resultado es cierto. Es el teorema de Kuratowsky: Si G es aplanable, entonces G no “contiene”
a K5 ni a K3,3 . Sin embargo hay que interpretar lo de que no “contiene” de cierta manera;
por ejemplo, es claro que si a K5 le agregamos un vértice enmedio de una arista, la nueva
gráfica tampoco será aplanable a pesar de no contener a K5 ni a K3,3 . También, por ejemplo,
es claro que la gráfica de petersen dibujada aquı́ abajo no es aplanable pues si contrajéramos
a puntos las aristas que unen a los dos pentágonos obtendrı́amos K5 . No haremos preciso
aquı́ el sentido de no “contiene” ni probaremos el teorema.

61
La gráfica dual de una (multi/seudo) gráfica plana conexa G es (multi/seudo) gráfica

G construida a través de G poniendo un vértice por cada cara de G y tal que dos vértices de
G ∗ están unidos mediante una arista si, y sólo si, las caras correspondientes en G comparten
arista.

10.47 Observación. (a) Como vimos en la figura, aun cuando G sea simple, G ∗ no
necesariamente lo es.
(b) G ∗ también es aplanable porque por cada cara de G sólo hay un vértice y por cada
arista de G hay exactamente una arista de G ∗ , ası́ que, dentro de cada cara, las aristas se
pueden dibujar de manera que no haya intersección (salvo en el vértice que representa la
cara).

62
(c) La gráfica dual de G ∗ es G.
(d) La gráfica dual depende del encaje en el plano. Por ejemplo, las dos gráficas G1 y G2
dibujadas abajo son isomorfas, pero observemos que las dimensiones de las caras de G1 son
(5, 5, 4, 3), mientras que las de G2 son (6, 4, 3, 3), lo cual hace que en G1∗ no haya vértice de
grado 6, mientras que en G2 sı́ lo hay.

Como primera aplicación de lo anterior probaremos que los únicos poliedros regulares son
los platónicos.
Se puede abstraer la definición de poliedro de diversas maneras. Nos interesa aquı́ anali-
zarlos como gráficas y considerar algunas propiedades básicas, las cuales mencionamos aquı́:
Son gráficas conexas, planas (más bien las consideramos dentro de la esfera), cada cara está
definida por un ciclo de longitud al menos 3 y cada vértice tiene grado al menos 3. A una
gráfica que cumple estas propiedades le llamamos poliédrica.

10.48 Observación. La gráfica dual de una gráfica poliédrica también es poliédrica.

10.49 Teorema. Los únicos poliedros regulares son los platónicos.


Demostración. Supongamos que G es una gráfica poliédrica regular en la que el grado de
cada vértice es g y el número de aristas de cada cara es l. Sea V el número de vértices, A
el número de aristas y C el número de caras de G. Por la definición de gráfica poliédrica
tenemos que
g, l ∈ {3, 4, 5}.
Además,
Vg Cl
A= = .
2 2
Consideremos los distintos casos para g:
∗ Si g = 3, entonces 3V2 = A = Cl2
, de donde V = Cl
3
y en la fórmula de Euler Cl
3
− Cl
2
+C =
2 y, por tanto, C(6 − l) = 12. Para l = 3 se tiene C = 4, A = 6 y V = 4; éste es el caso del
tetraedro. Para l = 4 se tiene C = 6, A = 12 y V = 8; éste es el caso del cubo. Para l = 5
se tiene C = 12, A = 30 y V = 20; éste es el caso del dodecaedro.
∗ Si g = 4, entonces 4V2 = A = Cl2
, de donde V = Cl
4
, y en la fórmula de Euler Cl
4
− Cl
2
+C =
2, ası́ que C(4 − l) = 8. Aquı́ la única posibilidad es l = 3, y entonces C = 8, A = 12 y
V = 6; éste es el caso del octaedro.
∗ Si g = 5, entonces 5V2 = A = Cl2
, de donde V = Cl
5
, y en la fórmula de Euler Cl
5
− Cl
2
+C =
2, y tenemos C(10 − 3l) = 20. Aquı́ la única posibilidad es l = 3, y entonces C = 20, A = 30
y V = 12; éste es el caso del icosaedro. ♦

63
Ahora consideraremos otra aplicación importante: la coloración de mapas. El problema
consiste en lo siguiente: Dado un mapa (de paı́ses), ¿cuántos colores bastan para colorear
cada paı́s de manera que si dos paı́ses comparten una frontera entonces su color es distinto?
El problema traducido a gráficas es el siguiente: Dada una gráfica plana en la que no hay
vértices de grado 1, ¿cuántos colores bastan para pintar sus caras de manera que caras
adyacentes tengan distinto color? Se ha demostrado, mediante el uso de la computadora,
que 4 colores bastan. Nosotros lo probaremos aquı́ para 5 colores. Consideraremos el caso
dual (coloración de vértices es decir, función del conjunto de vértices en un conjunto cuyos
elementos se llaman colores), recordando que los lazos en la gráfica dual de una gráfica se
producen cuando un vértice de la gráfica original tiene grado 1. Decimos que una coloración
de vértices es buena o propia si vértices adyacentes tienen distinto color.

10.50 Proposición. Sea G una gráfica plana sin lazos. Entonces existe una buena
coloración con 5 colores para los vértices de G.
Demostración. Sin pérdida de generalidad podemos suponer que la gráfica es conexa.
También es claro que podemos suponer que no hay aristas múltiples. Supongamos entonces
que G es una gráfica simple conexa. Procedemos por inducción sobre V . Si V = 1 no hay
nada que probar. Supongamos que V > 1 y que el resultado es cierto para gráficas con menos
de V vértices. Si la gráfica no es poliédrica, por ser simple y conexa debe de tener un vértice
de grado menor que 3; si la gráfica es poliédrica entonces, tiene un vértice de grado menor
o igual que 5; en cualquier caso, sea v un vértice de grado menor o igual que 5 y usemos la
hipótesis de inducción para colorear los vértices de G − v con 5 colores de manera que no
haya vértices adyacentes con el mismo color. Es claro que si alguno de los 5 colores no se usó
en los vértices adyacentes a v, entonces podemos pintar v con uno de ellos, obteniendo ası́
la buena coloración buscada para G. Veamos que existe una coloración para los vértices de
G − v en la que en los vértices adyacentes a v no se usan los 5 colores. Supongamos que no
y sean v1 , v2 , . . . , v5 los vértices adyacentes a v, escritos en orden de las manecillas del reloj;
sea i el color usado para vi . Por nuestra suposición, el color 3 no puede usarse para v1 (pues
v3 lo usa); eso quiere decir que v1 está unido a algún otro vértice ya pintado con el color 3,
pero análogamente, ese otro vértice no puede cambiarse de color al 1 por estar unido a otro
con el número 3 y ası́ sucesivamente; la única posibilidad de que no pueda intercambiarse el
color 1 por el 3 en toda la gráfica sin cambiar también el color de v3 es que se forme una
trayectoria de v1 a v3 en G − v en que los vértices vayan alternando los colores 1 y 3. Esto

64
mismo puede hacerse con los vértices v2 y v4 ; sin embargo esto produce dos trayectorias que
se intersectan; como la gráfica es plana, la intersección es un vértice común a los caminos,
lo cual es un absurdo.
1 3
...•
. .... .... .... .... .... .... ....• ....
....
.... ....
....
.... ....
.. 1
. ....
... ....
.. •
.
... . . .. .. .... ... . .... .... .... .......•
.
... .. .. .....
. . .. ...
....
.
...
...
v2... .... .... ...
. ...
4 .
... 2 ...
...
3 •........ •.......... .
... 2 ...
..
..
...
...
.. ... .. ...
..
.
...
...
... 3 ..•
.....
.
.
•....... 4
...
...
...
. .
........ .
............
v 3 ..
... .... ...... ...
......
v1 •....................... ...
...
... ..
..
.......
......... ...
...
..
1 ..................................................................... ...

.
•....................................
... .
... .... . • 2
... .
..

... v
.. ..........
..
. ..........
.......... 4
.......... .... ....
....
.
...

..
...
.
..
.

.......

...
..
.
v 4
v5 •.... 5

10.51 Ejercicio. (∗) Explicar por qué no puede continuarse la demostración de 10.50
para que, una vez cambiado el color de v3 por el color 1, continuar con un argumento similar
para intentar cambiar el color de v5 por el de v3 (o el de v4 por el de v2 ), y entonces concluir
que 4 colores son suficientes.

10.52 Ejercicio. (∗) Encontrar una gráfica plana sin lazos cuyos vértices no tengan
una buena coloración con 3 colores.

65
Referencias y lecturas complementarias
Bóna, M., A Walk through Combinatorics, World Scientific, 2002.

Bondy J.A., Murty, U.S.R., Graph Theory with Applications, The Macmillan Press Ltd,
1977.

Engel, A., Problem-Solving Strategies, Springer, 1997.

Lovász, L., Combinatorial Problems and Exercises, AMS Chelsea Publishing, American
Mathematical Society Providence Rhode Island, 2nd edition, 2007.

Pérez Seguı́ M. L., Combinatoria, Cuadernos de Olimpiadas de Matemáticas, Instituto


de Matemáticas, UNAM, 3a edición, 2009.

Pérez Seguı́ M. L., Combinatoria Avanzada, Cuadernos de Olimpiadas de Matemáticas,


Instituto de Matemáticas, UNAM, 2010.

Soulami, T.B., Les Olympiades de Mathématiques, Reflexes et Strategies, Ellipses Édi-


tion Marketing S.A., 1999.

Vilenkin, N., ¿De Cuántas Formas? (Combinatoria), Editorial Mir, Moscú, 1972.

66

También podría gustarte